🔙 Quay lại trang tải sách pdf ebook Tạp Chí Epsilon Số 20 Ebooks Nhóm Zalo CÔNG THỨC HARDY - RAMANUJAN QUA MÔ HÌNH CHẤT RẮN DEBYE Đàm Thanh Sơn ĐẠT ĐƯỢC SỰ ĐỒNG THUẬN TRÊN CÁC MẠNG NGẪU NHIÊN - QUYỀN LỰC CỦA SỐ ÍT Vũ Hà Văn & Trần Hoàng Bảo Linh VÀ CÁC CHUYÊN MỤC KHÁC NO 20 13AUG21 “… với độ giãn cách này thì xác suất lây nhiễm không bù lại được với xác suất virus bị chết do kháng thể của người sinh ra. Dịch vì thế phải hết.” Nguyễn Lê Anh (Đỉnh dịch?) "Trong tất cả các hình phẳng có chu vi bằng nhau thì hình tròn có diện tích lớn nhất." Trịnh Đào Chiến (Câu chuyện về bài toán đẳng chu trong hình học phẳng) Biên tập viên: Lê Viết Ân Võ Quốc Bá Cẩn Trần Quang Hùng Nguyễn Văn Huyện Lê Phúc Lữ Tống Hữu Nhân Nguyễn Tất Thu Đặng Nguyễn Đức Tiến Chủ biên: Trần Nam Dũng LỜI NGỎ Epsilon đã đi được một chặng đường gần 7 năm, và chúng tôi đã tiến tới cột mốc 20! Vào những ngày khởi đầu của Epsilon, phần lớn chúng tôi đều chỉ là những chàng trai trẻ, ngày nay, phần lớn chúng tôi đều đã trở thành những ông bố. Tóc chúng tôi cũng đã bạc nhiều hơn, sức chúng tôi cũng đã mệt mỏi. Nhưng chúng tôi biết, với đam mê, chúng tôi sẽ không dừng lại, với sự chia sẻ, chúng tôi sẽ được tiếp sức, và chúng tôi vẫn luôn tin rằng đi nhiều người, ta sẽ đi rất xa.... Vâng, chúng tôi vẫn biết, cuộc chiến với Covid-19 vẫn đang còn đó, chúng ta vẫn đang ở trong thời khắc khó khăn nhất. Nhưng, cho dù cuộc chiến sẽ còn cam go, vất vả, nhưng chúng ta vẫn luôn lạc quan tiến về phía trước. Và dường như những hoàn cảnh khắc nghiệt chưa bao giờ là rào cản cho những sáng tạo, cho những phát minh, cho những tìm kiếm mới. “Học nhi tri – Hành nhi tri – Du nhi tri – Khốn nhi tri”. Học để biết, hành để biết, giao du để biết và trải qua gian khó để biết. Không có điều kiện giao du trực tiếp, chúng ta có thể giao du trên mạng. Khó khăn vì covid-19 lại tạo cho những phương thức giao tiếp mới, những động lực sáng tạo mới."1. Per aspera ad astra.2 1Chúng tôi một lần nữa muốn lập lại thông điệp này từ Epsilon 19 2Hết cơn bĩ cực, tới hồi thái lai MỤC LỤC Đàm Thanh Sơn Công thức Hardy - Ramanujan qua mô hình chất rắn Debye . . . . . . . . . . . . . . . . 5 Vũ Hà Văn, Trần Hoàng Bảo Linh Đạt được sự đồng thuận trên các mạng ngẫu nhiên - Quyền lực của số ít . . . . . . . . . 9 Nguyễn Lê Anh Đỉnh dịch? . . . . . . . . . . . . . . . . . . . . . . . . . . . . . . . . . . . . . . . . . . 14 Nguyễn Lê Anh Vì sao không? . . . . . . . . . . . . . . . . . . . . . . . . . . . . . . . . . . . . . . . . 18 Lê Phúc Lữ, Nguyễn Nam, Đào Trọng Toàn Tuyển chọn các bài toán số học thi tuyển sinh 10 Chuyên 2021 . . . . . . . . . . . . . . 21 Tạ Hồng Quảng, Nguyễn Văn Huyện Bổ đề hoán vị (Phần 2) . . . . . . . . . . . . . . . . . . . . . . . . . . . . . . . . . . . 39 Võ Thịnh Phát Các bài toán tập hợp có nhiều điều kiện . . . . . . . . . . . . . . . . . . . . . . . . . . 52 Trần Nam Dũng Bất đẳng thức và cực trị trong đề thi tuyển sinh vào lớp 10 năm 2021 . . . . . . . . . . . 64 Võ Quốc Bá Cẩn Một số bài toán sử dụng phương pháp dồn biến toàn miền . . . . . . . . . . . . . . . . . 82 Ngô Văn Thái Mở rộng bài toán bất đẳng thức trong IMO - 2020 . . . . . . . . . . . . . . . . . . . . . 97 Trần Quang Hùng, Trương Tuấn Nghĩa, Đặng Minh Ngọc Một số ứng dụng nâng cao của phương tích . . . . . . . . . . . . . . . . . . . . . . . . 102 Trịnh Đào Chiến Câu chuyện về bài toán đẳng chu trong hình học phẳng . . . . . . . . . . . . . . . . . . 117 Ban Biên tập Bài toán hay - Lời giải đẹp . . . . . . . . . . . . . . . . . . . . . . . . . . . . . . . . . 130 4 Tạp chí Epsilon, Số 20, 08/2021 CÔNG THỨC HARDY-RAMANUJAN QUA MÔ HÌNH CHẤT RẮN DEBYE Đàm Thanh Sơn Chicago, Mỹ Bạn nào đã xem cuốn phim The Man Who Knew Infinity chắc sẽ nhớ một công thức đóng vai trò rất quan trọng trong phim: Công thức về hàm phân hoặch số nguyên, được Hardy và Ramanujan tìm ra năm 1918: Hàm phân hoạch p.n/ định nghĩa rất đơn giản. Lấy ví dụ số 4; số này có thể biểu diễn bằng 5 cách khác nhau thành tổng các số nguyên: 4 D 4 4 D 3 C 1 4 D 2 C 2 4 D 2 C 1 C 1 4 D 1 C 1 C 1 C 1 Như vậy p.4/ D 5: Tương tự p.5/ D 7; p.10/ D 42: Nhưng khi n tăng cao thì p.n/ tăng lên rất nhanh, ví dụ, p.200/ D 3:972:999:029:388: Trong phim MacMahon tính con số này bằng tay, không rõ bằng phương pháp nào. Hardy và Ramanujan tìm ra công thức cho tiệm cận của p.n/ với n lớn p.n/ 1 4np3exp r2n 3 ! : Nếu ta thay n = 200 vào công thức này thì ta sẽ tìm được p.200/ D 4; 10 1012; sai số 3:2% so với kế quả chính xác. Khi n càng lớn thì sai số này càng bé. Có vẻ phương pháp mà Hardy và Ramanujan dùng để tìm được công thức này khá phức tạp. Trong bài này chúng ta sẽ dùng vật lý để tiếp cận công thức Hardy-Ramanujan. Tìm được toàn bộ tiệm cận của p.n/ thì hơi khó, ta sẽ chỉ nhắm vào phần quan trọng nhất, phần exp thôi. Nói cách khác, chúng ta sẽ chứng minh: ln p.n/ r2n 3: Để tìm được công thức này, chúng ta sẽ dùng một cách tiếp cận không chính quy. Ta sẽ dùng mô hình Debye của nhiệt dung của chất rắn. Công trình này của Debye được viết năm 1912; vài năm trước khi Hardy và Ramanujan công bố công thức cho p(n). Có lẽ Hardy và Ramanujan không biết về công trình của Debye. 5 Hình 1: Một cảnh trong phim Tạp chí Epsilon, Số 20, 08/2021 Trước Debye người ta đã biết định luật Petit-Dulon, theo đó nhiện dung của một khối chất rắn là một hằng số không phụ thuộc vào nhiệt độ. Tuy nhiên thí nghiệm cho thấy định luật Petit-Dulon chỉ đúng ở nhiệt độ đủ cao, định luật này bị vi phạm ở nhiệt độ thấp. Einstein là người đầu tiên chỉ ra mối liên hệ giữa sự vi phạm định luật Petit-Dulon với cơ học lượng tử. Trong mô hình của Einstein, nhiệt dung là hằng số nếu nhiệt độ cao nhưng tiến tới 0 khi nhiệt độ giảm tới 0: Tuy nhiên trong mô hình của Einstein nhiệt dung tiến tới 0 nhanh hơn so với đo được trong thực nghiệm. Năm 1912 Debye đưa ra mô hình giải thích được sự biến thiên của nhiệt dung của chất rắn. Cách tiếp cận của Debye hết sức mới mẻ. Debye không nhìn chất rắn như một tập hợp các nguyên tử, ông nhìn chất rắn là một khí tạo ra bởi các hạt phonon - lượng tử của sóng âm thanh. Trong mô hình Debye, các nguyên tử chỉ là cái nền cho các hạt phonon lan truyển. Để liên hệ với công thức Hardy-Ramanujan ta chỉ cần xem xét một chất rắn 1 chiều. Để dễ tưởng tượng, ta sẽ xét một chiếc dây đàn, căng giữa hai điểm A và B: Ta chọn trục x của hệ toạ độ chạy theo đường thẳng nối hai điểm A và B: Nếu độ dài dây đàn là L thì tại A ta chọn x D 0; tại Bx D L: Khi ta gẩy đàn sẽ có sóng lan truyền trên dây đàn. Sóng này coi như là âm thanh trong môi trường một chiều. Để cho đơn giản ta giả sử dây đàn chỉ dao động theo chiều y. Trạng thái của dây đàn tại một thời điểm nào đó được mô tả bới hàm y D y.x/: Giả sử vận tốc lan truyền của sóng là v: Do hai đầu dây đàn bị đóng cứng, sóng trên dây đàn phải là sóng đứng, và biên độ của sóng biến thiên theo toạ độ và thời gian theo công thức y DX1 kD1 Ak cos.k!1t C ˛k/sin k xL : Trong công thức trên !1 là tần số cơ bản của dao động của dây đàn !1 D vL; và các hoạ ba (harmonic) cao hơn có tần số !k D k!1 với k D 2; 3; : : : Bây giờ ta lượng tử hoá cái dây đàn. Mỗi tần số !k nay tương đương với một dao động tử điều hoà, và dây đàn là một tổ hợp các dao động tử điểu hoà với tần số !1; !2; v.v. Các mức năng 6 Tạp chí Epsilon, Số 20, 08/2021 lượng của dao động tử điều hoà với tần số ! là ~!n C 12 : Như vậy để mô tả trạng thái lượng tử của dây đàn, ta cần một số vô hạn các số lượng tử n1; n2; : : : ; nk; : : : trong đó nk là số lượng tử của dao động tử với tần số !k D k!1: Như vậy E D E0 CX1 kD1 ~k!1nk; trong đó E0 là năng lượng của trạng thái cơ bản. Để đơn giản từ nay ta sẽ đo năng lượng của dây đàn từ E0; tức là cho E0 D 0: Bây giờ có thể nhận ra một điều như sau: Có p.n/ trạng thái lượng tử của dây đàn với năng lượng n~!1: Đây chính là điểm liên hệ giữa vật lý và công thức Hardy-Ramanujan. Nghĩ một lúc các bạn sẽ thấy điều này hầu như là hiển nhiên. Ví dụ ở mức năng lượng 4~!1 có năm trạng thái: n4 D 1; nk D 0; k ¤ 4 n3 D n1 D 1; nk D 0; k ¤ 1; 3 n2 D 2; nk D 0; k ¤ 2 n2 D 1; n1 D 2; nk D 0; k ¤ 1; 2 n1 D 4; nk D 0; k ¤ 1 Khi đã biết số trạng thái có năng lượng n~!1 bằng p.n/; ta kết luận ln p.n/ chính là entropy khi năng lượng bằng n~!1, theo định nghĩa của entropy qua tập thống kê vi chính tắc (micro canonical ensenble). Nhưng trong vật lý thống kê, ta có thể dùng tập thống kê chính tắc (canonical ensemble) để tính entropy của dây đàn, thay vì dùng vi chính tắc. Bình thường tính toán dùng tập thống kê chính tắc bao giờ cũng đơn giản hơn là dụng tập vi chính tắc. Một điểm làm đơn giản bài toán là khi nhiệt độ lớn hơn tần số dao động cơ bản, ta có thể bỏ qua hiệu ứng bề mặt của hai đầu dây đàn. Bây giờ dây đàn có thể coi là một chất khí phonon một chiều. Bài toán như vậy được đưa về dạng một chiều của bài toán mà Debye đã giải quyết năm 1912 khi ông tính được nhiệt dung của chất rắn ở nhiệt độ thấp. 7 Tạp chí Epsilon, Số 20, 08/2021 Bạn có thể làm tiếp những tính toán còn lại nếu bạn nào đã học vật lý thống kê; coi như đây là bài tập cho bạn. Bạn có thể tính entropy trực tiếp, hoặc tính nhiệt dung rồi lấy tích phân để tìm entropy. Kết quả là mô hình Debye của chất rắn cho ta phần exponent của công thức Hardy-Ramanujan. ln p.n/ r2n 3: Cách tiếp cận vật lý cho công thức Hardy-Ramanujan trên đây có trong cuốn B. Zwiebach, A First Course in String Theory. 8 Tạp chí Epsilon, Số 20, 08/2021 ĐẠT ĐƯỢC SỰ ĐỒNG THUẬN TRÊN CÁC MẠNG NGẪU NHIÊN QUYỀN LỰC CỦA SỐ ÍT Vũ Hà Văn, Trần Hoàng Bảo Linh GIỚI THIỆU Chiều 24=7; GS Vũ Hà Văn có bài báo cáo ở Viện nghiên cứu cao cấp về toán (VIASM) về một kết quả mới của anh và học trò là Trần Hoàng Bảo Linh. Do hội nghị được tổ chức dưới hình thức online nên tôi (Trần Nam Dũng) may mắn được nghe bài báo cáo này. Trong một tiếng đồng hồ, Vũ Hà Văn bằng một giọng nói dí dỏm, hài hước đã trình bày rất dễ hiểu về một vấn đề thú vị: Làm thế nào để đạt được sự đồng thuận. Nhận thấy rằng một chủ đề như vậy sẽ lôi cuốn các bạn trẻ yêu toán, tôi quyết định “dựng” lại bài nói chuyện của GS Văn theo trí nhớ và những ghi chép của mình. Tôi cũng có liên hệ với Trần Hoàng Bảo Linh để xin lại bản nháp của bài báo, chủ yếu để không có những sai sót trong những công thức quan trọng. Tiêu đề của bài viết này chính là tiêu đề của bài báo viết chung của Vũ Hà Văn và Trần Hoàng Bảo Linh, bài báo có phần tóm tắt như sau. Một cộng đồng gồm n cá thể chia thành hai phe, Đỏ và Xanh. Các cá nhân được kết nối bởi mạng xã hội, ảnh hưởng đến màu của họ. Mỗi người thay đổi màu mỗi ngày theo đa số hàng xóm của anh ấy/cô ấy. Đỏ (Xanh) thắng nếu mọi người trong cộng đồng trở thành Đỏ (Xanh) tại một thời điểm nào đó. Chúng tôi nghiên cứu quá trình này khi mạng cơ sở là đồ thị Erdos-Renyi ngẫu nhiên G.n; p/. Với trạng thái ban đầu cân bằng (n2người trong mỗi phe), rõ ràng là mỗi màu sẽ chiến thắng với xác suất như nhau. Nghiên cứu của chúng tôi cho thấy rằng đối với bất kỳ hằng số p và " nào, tồn tại một hằng số c sao cho nếu một phe có ít nhất n2 C c cá thể ở trạng thái ban đầu, thì sau đó phe đó sẽ thắng với xác suất ít nhất là 1 ". Điều đáng ngạc nhiên ở đây là c không phụ thuộc vào n, dân số của cộng đồng. Khi p D 12và " D 0:1, người ta có thể đặt c D 5, nghĩa là ban đầu một phe có n 2 C 5 thành viên. Nói cách khác, một phe chỉ cần thêm 5 người để giành chiến thắng trong một cuộc bầu cử với tỷ lệ cược áp đảo. Chúng tôi cũng tổng quát hóa kết quả thành p D pn D o.1/ trong một bài báo riêng. 9 Tạp chí Epsilon, Số 20, 08/2021 1. Quy tắc đa số (Majority rule) Xin giải thích quy tắc đa số qua ví dụ sau. Giả sử có n người thuộc về hai phe Xanh và Đỏ mà ta sẽ ký hiệu là B (Blue) và R (Red). Những người quen nhau (có kết nối xã hội) được nối bởi một đoạn thẳng. Mỗi ngày, họ sẽ đổi màu theo quy tắc số đông: Nếu số hàng xóm của một người (những người quen) có số đông màu gì thì họ sẽ đổi theo màu đó. Nếu có sự cân bằng về màu trong số các hàng xóm thì họ vẫn giữ màu như cũ. Sự cập nhật được xảy ra đồng loạt. Ta lấy ví dụ n D 6 và trạng thái ban đầu như sau thì các ngày tiếp theo, các cá thể sẽ đổi màu theo quy tắc đa số như hình minh họa: Hình 1: và tiếp theo ta thấy sẽ có sự tuần hoàn. 10 Tạp chí Epsilon, Số 20, 08/2021 Ta hoàn toàn có thể viết code để giả lập quá trình này cho một đồ thị bất kỳ và một cách phân chia màu bất kỳ. Trong bài báo của các tác giả có đường dẫn đến một giả lập như vậy. 1 2. Đồ thị ngẫu nhiên Erdos-Renyi Trong lý thuyết đồ thị, mô hình Erdos – Rényi là một trong hai mô hình có liên quan chặt chẽ ˝ với nhau để tạo ra các đồ thị ngẫu nhiên hoặc sự phát triển của một mạng ngẫu nhiên. Chúng được đặt theo tên của các nhà toán học Hungary Paul Erdos và Alfréd Rényi, những người lần ˝ đầu tiên giới thiệu một trong các mô hình vào năm 1959; trong khi Edgar Gilbert đã giới thiệu mô hình kia cùng lúc và độc lập với Erdos và Rényi. Trong mô hình của Erd ˝ os và Rényi, tất cả ˝ các đồ thị trên một tập đỉnh cố định .n/ với một số cạnh cố định đều có khả năng như nhau .p/; trong mô hình được đưa ra bởi Gilbert, mỗi cạnh có một xác suất cố định tồn tại hoặc vắng mặt, độc lập với các cạnh khác. Các mô hình này có thể được sử dụng trong phương pháp xác suất để chứng minh sự tồn tại của các đồ thị thỏa mãn các thuộc tính khác nhau hoặc để cung cấp một định nghĩa chặt chẽ về ý nghĩa của một thuộc tính đối với hầu hết các đồ thị. Có hai biến thể có mối quan hệ chặt chẽ với nhau của mô hình đồ thị ngẫu nhiên Erdos–Rényi. ˝ Hình 2: Một đồ thị được sinh bởi mô hình nhị thức của Erdos và Rényi ˝ .p D 0:01/ Trong mô hình G.n; M /; một đồ thị được chọn ngẫu nhiên với xác suất bằng nhau từ tập hợp tất cả các đồ thị có n và M cạnh. Các đỉnh được coi là có nhãn, có nghĩa là các đồ thị thu được từ nhau bằng cách hoán vị các đỉnh được coi là khác biệt. Ví dụ, trong mô hình G.3; 2/; có ba đồ thị hai cạnh trên ba đỉnh có nhãn (một đồ thị cho mỗi lựa chọn của đỉnh giữa trong đường hai cạnh) và mỗi trong ba đồ thị này được bao gồm với xác suất 13: Trong mô hình G.n; p/; một đồ thị được xây dựng bằng cách kết nối các nút có nhãn một cách ngẫu nhiên. Mỗi cạnh được đưa vào biểu đồ với xác suất p; độc lập với mọi cạnh khác. Tương tự, xác suất để tạo ra mỗi đồ thị có n đỉnh và M cạnh là pM .1 p/C2nM : Tham số p trong mô hình này có thể được coi là một hàm trọng số, khi p tăng từ 0 đến 1; mô hình ngày càng có nhiều khả năng bao gồm các đồ thị có nhiều cạnh và ngày càng ít có khả năng bao gồm các đồ thị có ít cạnh. Đặc biệt, trường hợp p D 12tương ứng với trường hợp tất cả 2C2n đồ thị trên n đỉnh được chọn với xác suất bằng nhau. 1https://github.com/thbl2012/majority-dynamics-simulation 11 3. Quyền lực của số ít Tạp chí Epsilon, Số 20, 08/2021 Xét một đồ thị ngẫu nhiên G.n; p/: Nếu ban đầu hai phe có quân số bằng nhau (n2người mỗi phe) thì do tính đối xứng, xác suất để mỗi bên thắng sẽ bằng nhau và bằng q < 12(do còn khả năng hòa, tức quá trình sẽ dừng lại hoặc tuần hoàn mà không có bên nào áp đảo được bên kia). Câu hỏi đặt ra là, một phe cần phải “chiêu mộ” ít nhất bao nhiêu người của phe kia (ví dụ có c người của phe Xanh chuyển sang phe Đỏ) để đảm bảo giành thắng lợi (với xác suất cao, ví dụ 90%; hay 1 "). Một cách tự nhiên ta nghĩ rằng con số này sẽ là một hàm số phụ thuộc vào n; chẳng hạn là pn: Thế nhưng, rất ngạc nhiên là kết quả không phải như vậy. Số c hóa ra không phụ thuộc vào n mà chỉ phụ thuộc vào p và ": Định lý 1 (Quyền lực của số ít). Hãy xem xét quá trình (đa số) trên Gn; 12 : Giả sử rằng rằng phe Đỏ có ít nhất n2 C 5 đỉnh ở trạng thái ban đầu, trong đó n > 1000: Khi đó Đỏ trận thắng sau ngày thứ tư với xác suất ít nhất là 90%: Ý tưởng chứng minh định lý là quan sát điều gì sẽ xảy ra với phe Đỏ (phe đông hơn) sau ngày thứ nhất, rồi ngày thứ hai và các ngày tiếp sau. Vào ngày thứ nhất, số lượng các hàng xóm Đỏ và Xanh của mỗi nút v đều là đại lượng ngẫu nhiên nhị thức, có trung bình bằng n2 C c và n2 c tương ứng. Định lý giới hạn trung tâm khi đó suy ra rằng phần lớn khối lượng của chúng tập trung trong một khoảng độ dài ‚.pn/ xung quanh kỳ vọng tương ứng của chúng. Một khoảng con có độ dài không đổi trong khoảng sẽ có khối lượng ‚.n12 /: Do đó, người ta kỳ vọng rằng xác suất để số Đỏ vượt quá số Xanh (trong vùng lân cận cụ thể đó) là 12 C .n12 /: Vì vậy, chúng ta kỳ vọng rằng số nút Đỏ sau ngày đầu tiên là n2 C .pn/: Các ý tưởng trực quan này sau đó đã được hiện thực hóa bằng các bổ đề khá kỹ thuật (Bổ đề 9 trong [1]). Tiếp theo, chúng ta phân tích tình hình sau ngày đầu tiên. Rõ ràng, nếu người ta đổi màu sau ngày 1 và kiểm tra đồ thị, phân phối của nó không còn là G.n; p/: Do đó, chúng ta không thể áp dụng phương pháp tương tự trong việc chứng minh Bổ đề 9 cho những ngày sau đó. Thay vào đó, chúng ta sử dụng “đối số thu hẹp” lập luận rằng có khả năng phe Xanh sẽ đơn điệu thu hẹp lại thành rỗng, bất kể sự lựa chọn của các thành viên, do cấu trúc của G: Cốt lõi của lập luận thu hẹp của chúng ta là bất đẳng thức Hoeffding, một kết quả cổ điển về tổng các biến ngẫu nhiên độc lập. Vì là một bài báo cáo mang tính giới thiệu nên GS Văn không đi chi tiết vào các chứng minh kỹ thuật. Ông cũng thông báo rằng kết quả tổng quát cũng đã được Linh Trần và ông hoàn thành và trình bày trong một bài báo khác. Cuối buổi hội thảo, GS Văn đã cùng trao đổi về các biến thể cũng như các bài toán liên quan cùng các mô hình thực tế. Các bạn đọc quan tâm có thể tìm hiểu thêm trong bài báo [1] dưới đây. 12 Tạp chí Epsilon, Số 20, 08/2021 Tài liệu tham khảo [1] Linh Tran, Van Vu: Reaching a consensus on random networks: A power of few https://arxiv.org/abs/1911.10279. [2] https://en.wikipedia.org/wiki/Erdos-Renyi_model 13 ĐỈNH DỊCH? Nguyễn Lê Anh GIỚI THIỆU Tạp chí Epsilon, Số 20, 08/2021 Trong hơn một năm qua, thầy Nguyễn Lê Anh luôn đưa ra các dự đoán trước và rất chính xác về tình hình Covid-19 trên thế giới cũng như ở Việt Nam. Vậy làm thế nào để có thể dự đoán chính xác trong điều kiện thiếu thốn thông tin và mọi thứ còn quá mới như vậy? Trong số Epsilon 20 này, chúng tôi trân trọng giới thiệu với độc giả bài viết phổ thông của thầy Lê Anh cách thức hình thành dự đoán, trong đó quan trọng nhất chính là việc hình thành tư duy của dự đoán này. Phần đông mọi người tin theo lập luận không có đỉnh dịch vì "thả con virus ra lại bùng phát tiếp". Trước hết nói về đỉnh dịch. Cần phải có một định nghĩa về đỉnh dịch trước khi nói nó ở đâu. Đỉnh dịch không hẳn là thời điểm có nhiều ca nhiễm nhất trong ngày, nó là cực đại của một đường cong dự báo. Vậy khi chưa có đường cong dự báo mà nói tới đỉnh dịch thì chỉ là loại "lang băm". Vậy có hay không một đường cong dự báo? Vào thời điểm của đợt dịch đầu tiên, gần như mọi người đều đã quên, khi ấy tất cả đều rất hoang mang, và không một ai nghĩ tới việc dịch sẽ kết thúc. Tôi thì không. Tôi không tin các thông tin trên các phương tiện thông tin đại chúng. Tôi lần tìm những bài báo khoa học. Khi ấy còn có rất ít báo cũng như hầu như trên khắp thế giới chưa có ai được coi là chuyên gia. Tôi bắt đầu từ việc phân tích các dữ liệu của đợt dịch Vũ Hán. Tôi cần phải biết là có hay không dịch ở Vũ Hán. Tôi suy nghĩ mãi và nhận thấy, Trung Quốc phong tỏa toàn bộ Vũ Hán. Vậy cái gì chứng minh là Trung Quốc đang phong tỏa Vũ Hán? Khi ấy tôi đang nghiên cứu về nguồn gốc bụi ở Hà Nội. Tôi chợt nhận ra bụi từ các tỉnh quẩn về Hà Nội rất nhiều. Tiếp theo tôi nhận ra một lượng lớn bụi từ đâu tới và chiếm ở tầng cao. Tôi lặng lẽ theo dõi bụi từ Trung Quốc ngay từ tháng 10 năm 2019. Đứng trước câu hỏi, "Có đúng là Trung Quốc đang phong tỏa Vũ Hán hay không?" tôi lặng lẽ đi kiểm tra tình trạng ô nhiễm ở khu vực này. Tôi giật mình khi thấy một vùng rộng tới 1000km quanh Vũ Hán không có bụi mịn trong nhiều ngày. Vậy đúng là các hoạt động sản xuất đang bị đóng băng. Tôi hiểu là dịch Corona là có thật. Tôi đi tìm các bài báo. Hầu như không có bài báo nào vào thời điểm Vũ Hán đang có dịch. Tôi tìm được giải mã gen của con Corona, khoảng 30kb. Tôi định đi phân tích cơ chế hoạt động của nó, nhưng khi ước tính thời gian để làm việc này lên tới 2 năm. Thế là tôi quyết định thôi và chuyển sang làm mô hình kiểu như vật lý. Lúc ấy tôi chưa hề biết gì về sự tồn tại của cái gọi là hệ phương trình dịch tễ. Tôi suy nghĩ cơ thể sẽ phải tự khỏi sau một số ngày. Sau khi cho Excel tính thô thì thấy dịch tuy có tăng nhanh nhưng sẽ kết thúc rất nhanh. Vậy là có đỉnh dịch, và không nhất thiết là cả cộng đồng bị lây nhiễm hết. Tôi còn nhớ có những trận cháy rừng mà vẫn có rất nhiều cây không bị 14 Tạp chí Epsilon, Số 20, 08/2021 cháy, vậy dịch bệnh cũng thế. Tôi lập trình mô phỏng dựa theo phân tích quá trình lây nhiễm theo quan hệ công việc, quan hệ gia đình, và quan hệ ngẫu nhiên. Kết quả là tôi nhận ra đồ thị của nó na ná như đồ thị lây nhiễm của SAR. Tôi cho rằng tính toán chỉ cần đúng 10% là tốt lắm, vậy nên tôi đi tìm công thức ốp vào được dạng đồ thị do máy tính ra. Có một điều nhiều bạn không biết, khoa học không phải là nói ra những câu tù mù. Phần quan trọng nhất của một lý thuyết khoa học là làm thể nào đơn giản hóa được sự phức tạp mà vẫn bảo đảm được độ chính xác cần thiết. Nếu các bạn mô tả dịch bệnh thông qua 100 tham số thì cái ấy khó có thể được coi là khoa học. Tôi mô tả sự lây nhiễm chính xác tới mức 10% mà chỉ cần 2 thông số. Vậy tôi tin là mình đang nắm được bản chất của vấn đề. Tôi cần phải tìm 2 tham số dựa trên các dữ liệu thực tế. Chắc chắn không thể có mô hình nào tính ra đươc đồ thị lây nhiễm, bởi họ cần quá nhiều tham số, và các tham số ấy là do họ "phịa" ra. Các dữ liệu tự nó không đúng do thời điểm bị nhiễm bệnh không thật sự rõ ràng. Như thế tôi càng tin là các mô hình không phải là của tôi đều không dùng được. Tôi có dạng đường cong và tôi chỉ vệc ốp nó để lấy thông số. Và tôi đã làm được điều ấy. Có điều tôi phải có đủ dữ liệu. Vậy tôi cần bao nhiêu dữ liệu? Do dữ liệu rất không rõ ràng nên tôi phải có một vài tuần dữ liệu. Tôi cũng đã có thể tìm cách giải phương trình dịch tễ. Trên thế giới nhiều người đi giải phương trình này. SIR1là sự chi li hóa phương trình Volter về cá. Nó là hệ phương trình có các tham số là hằng số. Có thể dùng phương pháp số để giải phương trình này. Để giải phương trình này thì chỉ cần thay đạo hàm bằng sai phân từ hiện tại tới quá khứ. Tuy nhiên như thế thì sai số tích lũy sẽ lớn. Người ta thay đạo hàm bằng sai phân nhưng lấy từ tương lai trở lại. Vậy là phải giải hệ phương trình tuyến tính ma trận thưa. Muốn đạt được độ chính xác cao hơn người ta phải dùng công thức xấp xỉ đạo hàm tốt hơn. Nói như vậy để bạn nào thích thì tự mà làm chứ đừng bấm bấm mấy cái trình có sẵn. Tuy nhiên, việc giải được hệ phương trình SIR chính xác chỉ là một trò vui toán học, thực tế cần cái khác. Chúng ta muốn biết dịch thực sự sẽ thế nào? Như tôi đã nói tôi cảm thấy họ đường con 2 tham số của tôi là đủ tốt, vậy tôi cho là nó phải là nghiệm của SIR. Tuy nhiên SIR thì không có lời giải giải tích đơn giản, vậy thì làm thế nào? Vấn đề là ở chỗ muốn SIR nhận cái họ hàm của tôi là nghiệm thì cần 2 yếu tố. Yếu tố thứ nhất các hệ số của SIR không phải là hằng số. Cái này thì không sao, miễn nó gần như là hằng số là ổn. Yếu tố thứ hai là lời giải chỉ cần gần với họ nghiệm của tôi là được. Vậy là là ổn. Họ hàm của tôi chỉ cần 2 tham số nhưng lại là nghiệm gần đúng của SIR. Tôi còn cần phải đưa ra lý giải vì sao lại chỉ cần 2 tham số. Tôi suy nghĩ mãi và phát hiện ra một điều, tôi cần phải biết thực trạng của bệnh dịch trong tương lai thực chứ không phải là mô hình giả định. Rõ ràng là dịch phụ thuộc nhiều vào các sự kiện và các phản ứng của chính quyền. Phản ứng của chính quyền thì tôi chưa thể biết. Tôi cũng chưa thể biết người dân Việt Nam này họ nghĩ ra ... những trò ma mãnh gì. Tuy nhiên tôi chợt nhận ra là con người chỉ có thể làm chủ khi anh ta còn chưa bị đe dọa trước cái chết. Đứng trước nỗi sợ hãi của cái chết ai cũng như ai, phải tuân thủ theo các quy luật nhất định nào đấy. Tôi gọi nó là văn hóa. Văn hóa là các nguyên lý định hướng tư duy của một cộng đồng dân cư. Chức năng lớn nhất của văn hóa là duy trì sự tồn tại. Vậy là tôi chạm vào mô hình văn hóa. Ở đây là văn hóa giao tiếp. Văn hóa thì thay đổi rất ít, ngay cả hệ thống lãnh đạo cũng xuất hiện trên nền văn hóa dân tộc, vậy thì tôi không cần phải biết chính quyền sẽ ra các sắc lệnh gì, tôi chỉ cần có một mô hình về sự lây nhiễm. Tôi cho rằng có một khoảng giãn cách xã hội mà theo đó nếu lớn hơn thì nền kinh tế sẽ suy sụp, và nếu nhỏ hơn thì dịch bệnh sẽ bùng phát. Rõ ràng là một khi khoảng giãn cách ở một khu vực nào đó mà nhỏ thì dịch bùng phát. Khi ấy chính quyền phải tìm cách làm cho khoảng 1SIR (Susceptible – Infectious – Recovered) là một mô hình toán học thường được dùng trong dịch tễ học. 15 Tạp chí Epsilon, Số 20, 08/2021 giãn cách lớn thêm ra. Tôi lập mô hình game với sự tấn công làm giảm độ giãn cách và thời gian phản ứng để làm cho nó quay trở lại độ giãn cách ban đầu. Với độ giãn cách này thì xác suất lây nhiễm không bù lại được với xác suất virus bị chết do kháng thể của người sinh ra. Dịch vì thế phải hết. Vậy là tôi đi đến mô hình game, giống như bóng đá và nhận thấy dạng hàm của tôi mô 16 Tạp chí Epsilon, Số 20, 08/2021 Hình 1: Dự báo tình hình dịch vào ngày 13 tháng 8 năm 2021. tả đủ tốt trận đấu. Như thế các bạn có trình độ lùn đừng nghĩ là mọi thứ được tự do. Bất luận cái gì có trong thực tế cũng chịu sự ràng buộc nhất định. Và cũng chính vì vậy mà đường cong phản ánh được sự diễn biến thực của dịch. 17 VÌ SAO KHÔNG? Nguyễn Lê Anh GIỚI THIỆU Tạp chí Epsilon, Số 20, 08/2021 Liên tục trong 7 số Epsilon gần đây, chúng tôi luôn có chuyên mục phám phá lịch sử thông qua tư duy lô-gích và phương pháp suy luận khoa học. Chúng tôi muốn giới thiệu với độc giả chúng ta có thể làm được những gì với tư duy toán học, thông qua những nghiên cứu miệt mài của nhà toán học Nguyễn Lê Anh. Trong số này, chúng tôi tiếp tục đăng những ghi ghép của ông, những nổ lực không ngừng nghỉ cùng với những khám phá mới. Đây cũng là lời kêu gọi cộng đồng trong việc cùng chung tay xây dựng một tài liệu lịch sử cho dân tộc chúng ta - dân tộc Việt Nam. Tôi muốn biết về sắt bởi tôi muốn biết nguyên nhân vì sao Bà Trưng lại thua Mã Viện, mặc dù cũng tiêu diệt hơn một nửa tức hơn 10 nghìn quân của Mã Viện. Marx đã nói rất rõ là sức sản xuất có quan hệ tương ứng với quan hệ sản xuất. Và sức sản xuất thì được xác định thông qua công nghệ chế tác công cụ lao động. Thời kỳ đồ đá con người chế tác ra các loại vũ khi để đánh nhau với thú dữ. Khi ấy họ tồn tại dưới dạng bầy đàn. Đồ đồng giúp cho sức lao động tăng đến mức mỗi người có thể tạo ra được lượng đồ ăn nhiều hơn nhu cầu bản thân sử dụng. Đồ đồng là sự bắt đầu của chế độ chiếm hữu nô lệ. Nô lệ thì cũng như là con vật, không có đủ đồ ăn hay ốm yếu thì bị chủ giết bỏ. Các cuộc chiến tranh săn bắt nô lệ xảy ra liên miên. Không có hậu cần, các cuộc chiến không thể đi quá xa, nên lượng nô lệ bắt được giảm dần. Đồ sắt là sự bắt đầu của các loại vũ khí dài nhẹ mà sắc, chiếm ưu thế trong các cuộc chiến với vũ khí bằng đồng. Hơn thế sắt được dùng để làm trục xe kéo, dùng để đóng móng cho ngựa, tức là tạo ra sự chủ động hậu cần. Nhờ có đồ sắt mà các cuộc viễn chinh xuất hiện. Phạm vi các cuộc săn bắt nô lệ lớn dần, lượng nô lệ bắt về càng ngày càng nhiều. Xuất hiện các thành phố đông dân. Do đông dân mà phải cần tới luật pháp và từ đó sinh ra khái niệm quốc gia. Quốc gia để lại dấu ấn là các nền văn minh. Vậy để nhận ra được tầm cỡ của cuộc chiến 20 nghìn quân mỗi bên vào đầu Công Nguyên chúng ta phải đi tìm hiểu về kỷ nguyên đồ sắt. Ở Việt Nam, thời đại luyện kim đồng thau được cho là tiếp theo của văn hóa Phùng Nguyên (niên đại 4000-3500 năm cách ngày nay). Đối với đồ sắt, các phát hiện khảo cổ mới chỉ cho thấy có dấu hiệu khả nghi về sự tồn tại của nghề luyện sắt từ thế kỷ 2-3 trước Công Nguyên. Đó là những vết tích nghi là lò luyện sắt, hòn quặng sắt, xỉ sắt hình giọt nước tại di chỉ Đồng Mỏm thuộc xã Diễn Thọ huyện Diễn Châu tỉnh Nghệ An. Các hiện vật gỉ sắt không rõ hình dạng cũng được phát hiện tại di chỉ Đường Mây, Cổ Loa, Hà Nội. Tuy nhiên những dấu hiệu khả nghi này chưa đủ để khẳng định đã có công nghệ luyện kim sắt ở Bắc Bộ. Khẳng định về sự tồn tại của công nghệ luyện kim sắt là rất khó. Trước hết là bản thân sắt rất dễ bị oxy hóa. Ở khu vực nhiệt đới nóng ẩm mưa nhiều lại cạnh biển như Bắc Bộ, rất khó có thể tìm được một dụng cụ nào bằng sắt vào khoảng trước đầu công nguyên. 18 Tạp chí Epsilon, Số 20, 08/2021 Nhiều bạn chưa hiểu về cách ước lượng tuổi của các cổ vật. Một cái trống đồng thì chả thể nói tới việc tuổi của nó. Người ta tính tuổi của trống đồng thông qua các hiện vật có thể tính được tuổi đi kèm. Vì thế mà chả thể nói được tuổi của một cái trống vớt được ở lòng sông. Và cũng chả thể nói được tuổi của cái trống khi mà nó đã được chùi cọ sạch sẽ. Đối với đồ sắt có niên đại trước Công Nguyên nhiều khi chỉ còn lại một lớp đất có một ít gỉ oxit sắt. Để xác định được tuổi của nó người ta phải tìm hiểu các loại vi sinh vật dính ở đấy. Như thế các hiện vật khảo cổ bằng sắt chẳng những rất hiếm mà xác định chính xác tuổi lại còn khó hơn. Vậy chúng ta không nên hạn chế việc tìm hiểu thời đại đồ sắt trong một khu vực dân cư mà nên mở rộng ra với một không gian lớn hơn để nhìn ra sự di cư, nhìn ra sự trao đổi công nghệ. Để tìm hiểu về công nghệ chế tác sắt thời đại Bà Trưng thì nên bắt đầu nghiên cứu khảo cổ thời đại đồ sắt từ các quốc gia lân cận. Gỗ bắt đầu bốc cháy ở 300 độ C, và khi cháy nó tạo ra hơi cháy, tại nơi đó nhiệt độ lên đến 600 độ C. Các loại gạch gốm chỉ được nung ở tầm 600 độ C. Nó màu đỏ và mềm. Để có được gốm cứng như sứ thì phải nung được tới nhiệt độ khoảng 900 độ C. Nhiệt độ than củi có thể đạt đến 1000 độ C, vì thế đồ gốm có thể xuất hiện, một cách tình cờ, ở tất cả các cộng đồng người tiền sử. Nhìn vào màu sắc và độ cứng của gốm là chúng ta biết được nhiệt độ nung, biết được mức độ công nghệ của họ, và từ đây là mức độ văn minh. Muốn có được sứ thì đất sét phải được nung ở nhiệt độ cao hơn. Nếu để trong lò nung sứ một cục đồng hẳn là nó sẽ chảy ra, từ đây chúng ta có thể đồng nhất việc xuất hiện công nghệ luyện kim đồng với ghề gốm sứ. Tức tạo ra được nhiệt độ 1084.62 độ C. Nhiệt độ này phải được tạo ra một thời gian khá lâu, vậy lò nung phải lớn và được cung cấp đủ oxy. Vậy là phải có hoặc hệ thống khí phải thổi mạnh vào lò lớn, hoặc phải phải đốt bằng than đá. Thổi vào lò lớn có vẻ không khả thi nếu đúc các sản phẩm có độ tinh sảo cao, vậy nếu bề mặt của đồ đồng mà có các hoa văn trang trí thì hẳn là phải đổ dung dịch đồng vào khuôn và như thế lò phải được đốt bằng than đá. Nhiệt độ cháy ngay bề mặt cục than đá là 2500 độ C, nhưng nhiệt độ giảm nhanh theo số mũ, vậy nếu lò đốt mà nhỏ không tạo ra đủ năng lượng làm chảy sắt. Công nghệ nấu chảy đồng ngẫu nhiên phát hiện ra, nhưng nấu chảy sắt là không hề đơn giản. Cần phải tạo được nhiệt độ cao hơn 1538 độ C. Điều này không hề đơn giản, và là bí quyết công nghệ. Vậy đường đi của công nghệ luyện kim sắt thế nào, đến được Đại Việt vào khi nào? Công nghệ luyện kim sắt được cho là đã xuất hiện ở Trung Á và vùng đồng bằng Ấn Độ vào khoảng thời gian 3200 năm về trước. Công nghệ này đã lan truyền ra tới khu vực Tứ Xuyên, và có thể coi Xuân Thu Chiến Quốc là bắt đầu cuộc chiến tranh chinh phục của nền văn minh công nghệ đồ Sắt. Xuân Thu Chiến Quốc 771TCN đến 476TCN, và các cuộc đánh chiếm tiếp theo cho đến hết thời Tần, vào khoảng năm 210 TCN. Như vậy có thể là do chăn thả gia súc mà đã tạo ra sự giao tiếp Đông-Tây, công nghệ luyện kim sắt tới được vùng Tứ Xuyên vào khoảng 770 năm trước Công Nguyên. Phương pháp Thermoluminescence được Aitken phác thảo (1985, 1998) để áp dụng cho một hạt cát thạch anh. Theo nguyên lý, nếu tinh thể thạch anh ở lâu dưới ánh sáng mặt trời thì các điện tử bị vướng trong các lỗ hổng của nó sẽ nhận đủ năng lượng và thoát ra ngoài. Khi tinh thể bị che lấp dưới đất sâu, các điện tử tự do sẽ bị vướng vào các lỗ hổng thạch anh. Tinh thể sẽ có dư điện tích âm. Khi được làm nóng điện tử dư này sẽ được giải phóng ra và phát sáng. Đo số lượng hạt ánh sáng thoát ra này người ta sẽ phát hiện được thời gian hạt thạch anh bị vùi lấp sâu dưới đất. Phương pháp này rẻ tiền nhưng độ chính xác không cao. 19 Tạp chí Epsilon, Số 20, 08/2021 Tại các di chỉ khảo cổ ở Taungthaman thuộc Myanmar người ta tìm thấy lưỡi câu cá trong mộ chôn cùng người. Phương pháp xác định tuổi bằng Thermoluminescence cho kết quả mẫu vật 186a1 có tuổi khoảng 680BC-240BC. Vậy là luyện kim sắt đã di chuyển tới Myanmar vào khoảng từ 250 năm trước Công Nguyên. Tại Campuchia các di chỉ khảo cổ dọc theo sông Cửu Long cho thấy chúng thuộc văn hóa Óc Eo. Óc Eo là di chỉ ở núi Ba Thê huyện Thoại Sơn, tỉnh An Giang. Người ta đã tìm thấy nhiều loại tiền xu trong đó có tiền xu La Mã. Có tiền xu có hình Antoninus Pius và một bản sao của tiền xu Marcus Aurelius với một mặt để trống. Những đồng tiền La Mã cho thấy Óc Eo là một thương cảng lớn của vương quốc Phù Nam từ thế kỷ I đến thế kỷ VII. Gò Ô Chùa cách xa bờ biển hiện nay đến 150km. Qua nghiên cứu một số mẫu than tro bằng phương pháp cacbon C-14, kết quả cho thấy làng cổ gò Ô Chùa đã tồn tại cách ngày nay khoảng 3000 đến 2000 năm. Người ta đã tìm thấy ở gò Ô Chùa nhiều dụng cụ gốm làm muối giống như ở châu Âu vào thời kỳ 3000-2000 năm trước đây. Khoảng 20000 năm trước mực nước biển thấp hơn ngày nay 120 m, nhưng ở thời điểm 5000 năm trước, mực nước biển lại cao hơn đến 5m so với ngày nay. Sau đó, nước biển dần thấp xuống tới mực nước như ngày nay. Như thế gò Ô Chùa xưa là ven biển. Các nhà khảo cổ Việt - Đức đã Họ đã thực hiện khoan lấy mẫu trầm tích và khẳng định các lớp đất xung quanh địa điểm này là tầng trầm tích biển có niên đại bằng với trung tâm nấu muối Gò Ô Chùa. Nhà địa lý Hy Lạp Claudius Ptolemaeus đã sang phương Đông hồi đầu Công Nguyên bằng đường thủy, đã tả một nơi mà ông gọi là Kattigara. R.A. Stein thì cho rằng lời văn miêu tả phù hợp với khung cảnh Bình Trị Thiên, tuy nhiên đa số trong giới học giả cho Kattigara là Óc Eo. Như vậy để có thể hiểu ra được thời đại Bà Trưng chúng ta cần phải tìm hiểu cổ sử của không chỉ các quốc gia từ Trung Quốc cho tới Ấn Độ, các nước như Myanmar, Lào, Thái Lan, Campuchia, mà còn cả giao lưu giữa Đông và Tây. Vậy là chúng ta cần phải biết và nên dạy cho học sinh phổ thông vốn kiến thức về lịch sử và khảo cổ các nền văn minh từ các quốc gia trong khư vực và cả thế giới. Rõ ràng lịch sử của dân tộc chúng ta liên quan tới mọi ngóc ngách của thế giới, liên quan tới mọi sự kiện lớn xảy ra trên địa cầu này, trong hệ mặt trời này. Chủ nhân của một lịch sử hoành tráng như vậy thì dân tộc chúng ta đâu có thể là dân tộc hèn kém. Vì sao không nghiên cứu đến tận cùng để hiểu? 20 Tạp chí Epsilon, Số 20, 08/2021 TUYỂN CHỌN CÁC BÀI TOÁN SỐ HỌC THI TUYỂN SINH 10 CHUYÊN 2021 Lê Phúc Lữ (ĐH Khoa học tự nhiên TPHCM) Nguyễn Nam (THPT Chuyên Nguyễn Du, Đăk Lăk) Đào Trọng Toàn (THPT Chuyên Bến Tre) TÓM TẮT Bên dưới, nhóm tác giả có tổng hợp gần 100 bài toán Số học trong đề thi tuyển sinh THPT Chuyên của các tỉnh, khối chuyên trên cả nước. Có một số nơi mà trong đề thi không có ra dạng Toán này, nhưng có một số nơi khác thì ra khá nhiều. Nhìn một cách tổng quát, các đề thi đều tập trung vào phần phương trình nghiệm nguyên, là một khía cạnh dễ khai thác và cũng khá quen thuộc đối với học sinh THCS. Xin cám ơn các thầy cô, các bạn học sinh của group facebook “Hướng tới Olympic Toán VN” đã hỗ trợ nhóm tác giả hoàn thành tài liệu này. 1. Các bài toán có lời giải Bài toán 1 (Thái Bình). Giả sử n là số tự nhiên thỏa mãn n(n + 1) + 7 không chia hết cho 7. Chứng minh rằng 4n3 − 5n − 1 không là số chính phương. Lời giải. Theo đề bài thì n, n + 1 không chia hết cho 7. Ta có 4n3 − 5n − 1 = (n + 1)(4n2 − 4n − 1) và dễ thấy nếu gcd(n + 1, 4n2 − 4n − 1) = d thì có ngay d|7. Nếu d = 1, ta phải có cả hai số trên là số chính phương, nhưng điều này không thể do (2n − 2)2 < 4n2 − 4n − 1 < (2n − 1)2, ∀n ≥ 1. Do đó d = 7, kéo theo 7|n + 1, cũng vô lý. Bài toán 2 (Vũng Tàu). Tìm tất cả các cặp số nguyên (x, y) thỏa mãn (xy − 1)2 = x2 + y2. Lời giải. Ta viết lại x2y2 − 2xy + 1 = x2 + y2hay (x + y + xy)(x + y − xy) = 1. Từ đó ta suy ra hai thừa số cùng bằng 1 hoặc cùng bằng −1. Đến đây thì giải hệ là được các giá trị (x, y) = (0; ±1),(±1; 0). 21 Tạp chí Epsilon, Số 20, 08/2021 Bài toán 3 (Tây Ninh). Tìm nghiệm nguyên của x2 − 2y(x − y) = 2(x + 1). Lời giải. Ta viết lại phương trình thành 2y2 − 2xy + x2 − 2x − 2 = 0. Coi đây là phương trình theo biến y thì ta có ∆0 = x2 − 2(x2 − 2x − 2) ≥ 0 hay −1 ≤ x ≤ 5. Từ đó xét các trường hợp để thu được (x, y) = (0; ±1),(4; 1),(4; 3). Bài toán 4 (Hà Nam). Giải phương trình nghiệm nguyên x3 + y2 − x + 3z = 2021. Lời giải. Chú ý rằng x3 − x luôn chia hết cho 3, còn 2021 chia 3 dư 2 nên y2chia 3 dư 2. Đây là điều vô lý. Bài toán 5 (Cần Thơ). Tìm tất cả các cặp số nguyên (x, y) thỏa mãn x2 + 5y2 + 4xy + 2x + 4y − 3 = 0. Lời giải. Phương trình đã cho viết lại thành (x + 2y + 1)2 + (y − 2)(y + 2) = 0. Do đó (y − 2)(y + 2) ≤ 0 nên −2 ≤ y ≤ 2. Đến đây thử trực tiếp được y = 0, y = ±2. Bài toán 6 (Bình Phước). 1. Tìm nghiệm nguyên của (2x + y)(x − y) + 3(2x + y) − 5(x − y) = 22. 2. Cho hai số tự nhiên a, b thỏa mãn 2a2 + a = 3b2 + b. Chứng minh rằng 2a + 2b + 1 là số chính phương. Lời giải. 1) Phương trình đã cho viết lại thành (2x + y − 5)(x − y + 3) = 7. Xét các trường hợp có thể xảy ra, ta tìm được (x, y) = (−2; 8),(−2; 2). 2) Ta có (a − b)(2a + 2b + 1) = b2. Đặt d = gcd(a − b, 2a + 2b + 1) thì d2|b2 nên d|b. Suy ra d|a nên kéo theo d|2(a + b), từ đó có d|1 nên d = 1. Do đó, a − b và 2a + 2b + 1 là số chính phương. 22 Tạp chí Epsilon, Số 20, 08/2021 Bài toán 7 (Hà Nội). 1. Cho x là số thực khác 0 sao cho x +2xvà x3 đều hữu tỷ. Chứng minh rằng x hữu tỷ. 2. Tìm tất cả các cặp số (x, y) nguyên sao cho x2 + 5xy + 6y2 + x + 2y − 2 = 0. 3. Chứng minh rằng với mọi n nguyên thì n2 + n + 16 không chia hết cho 49. Lời giải. 1) Ta có x +2x 2= x2 +4x2+ 4 nên x2 +4x2 hữu tỷ, kéo theo x2 + 2 + 4x2 ∈ Q. Mặt khác, vì x3 ∈ Q nên 8x3 ∈ Q và x3 −8x3= x −2x x2 + 2 +4x2 ∈ Q. Từ đó có ngay x −2x∈ Q nên x hữu tỷ. 2) Ta viết lại phương trình thành (x + 2y)(x + 3y + 1) = 2. Từ đó xét các trường hợp để có (x, y) = (1; 0),(−2; 0),(6; −2),(3; −2). 3) Ta có n2 + n + 16 = (n + 4)(n − 3) + 28 nên để có 49|n2 + n + 16, ta cần có n + 4 hoặc n − 3 chia hết cho 7. Mà hai số này cách nhau 7 đơn vị nên chúng đều phải chia hết cho 7, khi đó dễ dàng có 28 chia hết cho 49, vô lý. Bài toán 8 (Vĩnh Phúc). 1. Cho các số nguyên x, y, z thỏa mãn x2 + y2 + z2 = 2xyz. Chứng minh rằng 24|xyz. 2. Tìm tất cả các bộ ba số nguyên (a, b, c) sao cho (a + b + c)2 − 2a + 2b là các số chính phương. Lời giải. 1) Từ giả thiết thì x2 + y2 + z2chẵn nên trong các số x, y, z, phải có ít nhất một số chẵn, giả sử là x. Khi đó, 4|x2 và 4|2xyz nên 4|y2 + z2, từ đó dễ thấy phải có y, z đều chẵn. Ngoài ra, nếu x, y, z đều không chia hết cho 3 thì x2, y2, z2chia 3 dư 1 nên 3|x2 +y2 +z2. Vì thế 3|2xyz, mâu thuẫn. Do đó 3|xyz, kết hợp với trên thì có 24|xyz. 2) Đặt M = (a + b + c)2 − 2a + 2b thì ta có (a + b + c)2 + 2(a + b + c) + 1 > M và (a + b + c)2 − 2(a + b + c) + 1 < M. Do đó M = (a + b + c)2và thay vào có ngay a = b. Vậy các bộ số nguyên cần tìm là a = b và c tùy ý. 23 Bài toán 9 (Lào Cai). Tạp chí Epsilon, Số 20, 08/2021 1. Tìm tất cả các bộ số nguyên (x, y) thỏa mãn x2 − 2x + 2y2 = 2(xy + 1). 2. Tìm số nguyên tố p lớn nhất sao cho tồn tại các số nguyên dương x, y thỏa mãn x3 + y3 − p = 6xy − 8. Lời giải. 1) Coi phương trình đã cho là bậc hai theo biến x, tính delta theo y và chặn được 1 ≤ y ≤ 3. Giải ra được (x, y) = (0; 1),(4; 1),(4; 3). 2) Ta viết lại (x + y + 2)(x2 + y2 + 4 − xy − 2x − 2y) = p. Vì x+y +2 > 1 nên cần có x2 +y2 +4−xy−2x−2y = 1 và x+y +2 = p. Thế y = p−2−x vào, ta được x2 + (p − 2 − x)2 + 4 − x(p − 2 − x) − 2x − 2(p − 2 − x) = 1 hay 3x2 − 3x(p − 2) + p2 − 6p + 11 = 0. Tiếp tục tính delta, ta thu được 4 ≤ p ≤ 8 nên p ≤ 7. Thử trực tiếp với p = 7 có (x, y) = (2, 3) thỏa mãn. Do đó, giá trị lớn nhất cần tìm là 7. Bài toán 10 (Phú Thọ). 1. Tìm tất cả các cặp số nguyên (x, y) thỏa mãn (x2y − xy + y)(x + y) = 3x − 1. 2. Tìm các số nguyên tố p, q thỏa mãn đồng thời: i. p2q + p chia hết cho p2 + q. ii. pq2 + q chia hết cho q2 − p. Lời giải. 1) Viết lại đề bài thành y(x2 − x + 1)(x + y) = 3x − 1 nên x2 − x + 1|3x − 1. Suy ra x2 − x + 1|(3x − 1)(3x − 2) = 9x2 − 9x + 2. Do 9x2 − 9x + 2 = 9(x2 − x + 1) − 7 nên đưa về x2 − x + 1|7. Suy ra x2 − x + 1 ∈ {1, 7}. Giải ra có ngay x = 0, x = 1. 2) Nếu p = q thì ta cần có p3 + p chia hết cho p2 + p hay p2 + 1 chia hết cho p + 1, kéo theo 2 chia hết cho p + 1, vô lý. Nếu p, q phân biệt, ta có gcd(p, p2 + q) = gcd(q, q2 − p) = 1 nên phải có p2 + q|pq + 1 và q2 − p|pq + 1. Từ p2 + q|pq + 1, ta có p2 + q ≤ pq + 1 nên q ≥ p + 1. Từ q2 − p|pq + 1, ta có q2 − p ≤ pq + 1 hay q(q − p) ≤ p + 1 ≤ q hay q − p ≤ 1. Do đó ta phải có q = p + 1 nên (p, q) = (2; 3). Thử lại các giá trị này, ta thấy thỏa mãn. 24 Tạp chí Epsilon, Số 20, 08/2021 Bài toán 11 (Vĩnh Long). 1. Chứng minh rằng tổng các bình phương của 6 số nguyên liên tiếp thì không thể là số chính phương. 2. Tìm các nghiệm nguyên dương của phương trình x2y + 2xy + y = 32x. Lời giải. 1) Trong 6 số nguyên liên tiếp thì có 3 số chẵn và 3 số lẻ, mà bình phương của số chẵn thì chia hết cho 4, trong khi bình phương của số lẻ thì chia 4 dư 1. Vì thế nên tổng các số đã cho chia 4 dư 3, không thể là số chính phương. 2) Ta viết lại đề bài thành y(x + 1)2 = 32x. Suy ra (x + 1)2|32x, mà gcd(x, x+ 1) = 1 nên ta có ngay (x + 1)2|32, từ đó (x + 1)2 = 4 hoặc 16 và giải ra được x = 1, x = 3. Bài toán 12 (Đăk Lăk). 1. Tìm tất cả các số tự nhiên n, k để n4 + 42k+1 là số nguyên tố. 2. Tìm tất cả các số nguyên dương x, y sao cho x4 − x2 + 2x2y − 2xy + 2y2 − 2y − 36 = 0. Lời giải. 1) Ta có hằng đẳng thức x4 + 4y4 = (x2 + 2y2)2 − 4x2y2 = (x2 − 2xy + 2y2)(x2 + 2xy + 2y2) nên từ đề bài, nếu đặt thì ta phải có p = n4 + 42k+1 = n4 + 4 · (2k)4 n2 − 2 · 2kn + 2 · 4k = 1 hay (n − 2k)2 + 4k = 1. Từ đây dễ dàng tìm được k = 0 và n = 1. Thử lại được p = 5 thỏa mãn. 2) Ta viết lại đề bài thành (x2 + y − 1)2+ (x − y)2 = 37 = 12 + 62 nên xét trực tiếp các trường hợp, ta có ngay (x, y) = (2; 3). Bài toán 13 (Tiền Giang). Cho m, n là các số nguyên dương thỏa mãn m2 + n2 + m chia hết cho mn. Chứng minh rằng m là số chính phương. 25 Tạp chí Epsilon, Số 20, 08/2021 Lời giải. Theo giả thiết thì m2 + n2 + m chia hết cho m nên m|n2. Đặt n2 = km với k ∈ Z. Khi đó m2 + n2 + m = m(m + k + 1) nên ta đưa về n|m + k + 1. Từ đây dễ thấy gcd(k, m) = 1 nên cả m, k phải đều là các số chính phương. Bài toán 14 (Kiên Giang). 1. Cho m, p, r là các số nguyên tố thỏa mãn mp + 1 = r. Chứng minh rằng m2 + r hoặc p2 + r là số chính phương. 2. Tìm tất cả các số nguyên tố q sao cho tồn tại n nguyên dương để n2 + 22q là lũy thừa với số mũ nguyên dương của 11. Lời giải. 1) Theo đề thì r > p và r > m. Nếu cả m, p đều lẻ thì kéo theo mp + 1 chẵn nên r chẵn, vô lý. Vì thế phải có m hoặc p chẵn. Giả sử m chẵn thì m = 2. Khi đó 2p + 1 = r và có p2 + r = (p + 1)2 là số chính phương. 2) Ta có n2 + 22q > 11 nên cần có n2 + 22q = 11kvới k ≥ 2. Từ đó dễ có 11|n. Thay vào suy ra 112|22q nên q = 11. Chọn n = 33 thì có 332 + 22 · 11 = 113thỏa mãn. Vì thế q = 11. Bài toán 15 (Đà Nẵng). Tìm tất cả các cặp số nguyên (x, y) thỏa mãn (x2 − y2)2 = 1 + 20y. Lời giải. Dễ thấy (x, y) là nghiệm thì (−x, y) cũng là nghiệm nên ta đưa về xét x ≥ 0. Ngoài ra, nếu y < 0 thì y ≤ −1 nên 1 + 20y < 0, không thỏa. Do đó, ta chỉ xét y ≥ 0. • Nếu (x − y)2 = 1 thì ta có ngay (x + y)2 = 1 + 20y nên giải hệ này là xong, thu được (x, y) = (±1; 0),(±5; 4),(±5, 6). • Nếu (x − y)2 > 1 thì (x − y)2 ≥ 4 và 1+20y ≥ 4(x + y)2 ≥ 4y2 và thu được 0 ≤ y ≤ 5. Kiểm tra trực tiếp thấy không thỏa. Bài toán 16 (Bình Thuận). 1. Gọi A là số tạo thành khi viết liên tục các số từ 1 đến 2021 thành một dãy liên tiếp, tức là A = 12345 . . . 20202021. Hỏi A có bao nhiêu chữ số và chữ số thứ 2021 của A tính từ trái là mấy? 26 Tạp chí Epsilon, Số 20, 08/2021 2. Cho p, x, y là các số tự nhiên thỏa mãn px2 + x = (p + 1)y2 + y. Chứng minh rằng x − y là số chính phương. 3. Cho a, b là các số nguyên dương mà a2 − ab +32b2chia hết cho 25. Chứng minh rằng a, b cùng chia hết cho 25. Lời giải. 1) Đếm số lượng chữ số của số có 1, 2, 3, 4 chữ số không vượt quá 2021, ta được A có tất cả 9 + 180 + 2700 + 4088 = 6977 chữ số. Tiếp theo, vì 9 + 180 < 2021 < 9 + 180 + 2700 nên chữ số cần tìm sẽ thuộc một số có 3 chữ số nào đó abc. Ta có 13(2021 − 9 − 180) = 610 23nên số cần tìm là chữ số thứ hai của số có 3 chữ số thứ 611 (tức là số 710). Vì thế, chữ số cần tìm là 1. 2) Ta viết lại đề thành (x − y)(p(x + y) + 1) = y2. Gọi d = gcd(x − y, p(x + y) + 1) thì d2|y2 nên d|y. Do đó d|x nên d|p(x + y) và kéo theo d|1 nên d = 1. Do đó, hai số x − y và p(x + y) + 1 là nguyên tố cùng nhau và vì tích của chúng chính phương nên bản thân mỗi số cũng phải là số chính phương. 3) Theo đề bài thì 2a2 − 2ab + 3b2chia hết cho 25. Ta có 2a2 − 2ab + 3b2 − 15b2 = 2(a − 3b)(a + 2b) nên suy ra a − 3b hoặc a + 2b chia hết cho 5. Tuy nhiên hai số này có hiệu là 5b chia hết cho 5 nên chúng cùng chia hết cho 5. Vì thế 25|15b2 nên 5|b, từ đó có ngay 5|a. Bài toán 17 (Cao Bằng). Tìm tất cả các cặp số nguyên (a, b) sao cho a4 − 2a3 + 10a2 − 18a − 16 = 4b2 + 20b. Lời giải. Ta viết lại đề bài thành (2a2 − 2a + 9)2= (4b + 10)2 + 45 hay (2a2 − 2a − 4b − 1)(2a2 − 2a + 4b + 19) = 45. Dễ thấy tổng của hai biểu thức trong hai dấu ngoặc là dương, mà tích của chúng cũng dương nên cả hai đều phải dương. Đến đây ta xét các trường hợp là được. Bài toán 18 (Nghệ An). 1. Tìm tất cả các số tự nhiên x, y sao cho x3 = 1993 · 3y + 2021. 2. Tìm tất cả số nguyên dương n sao cho n−23 n+89 là bình phương của một số hữu tỷ dương. 27 Tạp chí Epsilon, Số 20, 08/2021 Lời giải. 1) Nếu y = 0 thì x3 = 4041, không thỏa mãn. Nếu y = 1 thì có ngay x = 20. Xét y ≥ 2, ta viết lại phương trình thành x3 − 8000 = (x − 20)(x2 + 20x + 400) = 1993(3y − 3). Ta thấy vế phải chia hết cho 3 nhưng không chia hết cho 9. Tuy nhiên, từ 3|x3 − 8000, ta có x chia 3 dư 2 nên x − 20 chia hết cho 9, kéo theo vế phải cũng chia hết cho 9, vô lý. 2) Theo đề thì phải có (n − 3)(n + 89) là số chính phương. Ta đưa về (n + 33)2 − k2 = 3136 với k ∈ Z+. Từ đó xét các trường hợp để có n ∈ {32, 37, 73, 86, 167, 361, 752}. Bài toán 19 (Sóc Trăng). Tìm tất cả số nguyên dương n để n5 + n4 + 1 là số nguyên tố. Lời giải. Ta có phân tích n5 + n4 + 1 = (n2 + n + 1)(n3 − n + 1) nên một trong hai thừa số này phải là 1. • Nếu n2 + n + 1 = 1 thì có n = 0 hoặc n = −1, thử lại không thỏa. • Nếu n3 − n + 1 = 1 thì có n = 0, n = ±1 nên thử lại có n = 1 thỏa mãn. Bài toán 20 (Nam Định). 1. Tìm tất cả các cặp số nguyên dương (x, y) thỏa mãn x2y2(y − x) = 5xy2 − 27. 2. Cho p1, p2, . . . , p12 là các số nguyên tố lớn hơn 3. Chứng minh rằng p21 + p22 + · · · + p212 chia hết cho 12. Lời giải. 1) Ta viết lại đề bài thành xy2(x2 + 5 − xy) = 27. Do y2là ước chính phương của 27 nên y = 1 hoặc y = 3, từ đó thay vào được (x, y) = (1; 3). 2) Vì các số nguyên tố đã cho đều lớn hơn 3 nên không chia hết cho 3, suy ra bình phương của chúng chia 3 dư 1 và tổng bình phương của chúng sẽ đồng dư với 12 modulo 3, tức là chia hết cho 3. 28 Tạp chí Epsilon, Số 20, 08/2021 Bài toán 21 (Bình Dương). Cho các số nguyên a, b, c thỏa mãn a = b − c =bc. Chứng minh rằng a + b + c là lập phương của một số nguyên. Lời giải. Theo giả thiết thì b = a + c nên a + b + c = 2b. Ta có b − c =bcnên c(b − c) = b hay c2 − bc − b = 0 có nghiệm nguyên. Ta có ∆ = b2 + 4b là số chính phương. Đặt b2 + 4b = k2 với k ∈ N thì (b + 2)2 − k2 = 4 nên (b + 2 + k)(b + 2 − k) = 4. Chú ý rằng chênh lệch giữa hai thừa số là chẵn nên chúng chỉ có thể cùng là 2 hoặc cùng là −2, khi đó b + 2 + k = b + 2 − k nên k = 0 và b = 0 hoặc b = −4. Trong cả hai trường hợp, ta đều có 2b là lập phương đúng. Bài toán 22 (Lâm Đồng). 1. Cho a, b, c là các số nguyên thỏa mãn a + b + 20c = c3. Chứng minh rằng a3 + b3 + c3 chia hết cho 6. 2. Cho B = 2 + 22 + · · · + 22022. Chứng minh rằng B + 2 không là số chính phương. Lời giải. 1) Ta có x3−x = x(x−1)(x+1) chia hết cho 6 với mọi x ∈ Z. Suy ra (a3+b3+c3)−(a+b+c) chia hết cho 6. Chú ý rằng a + b + c = c3 − c − 18c chia hết cho 6 nên ta có 6|a3 + b3 + c3. 2) Ta có B2 = 1+2+· · ·+22021 nên B−B2 = 22022−1 hay B = 22023−2. Từ đó có B+2 = 22023 là một lũy thừa mũ lẻ của 2,không thể là số chính phương. Bài toán 23 (Bến Tre). Giải phương trình nghiệm nguyên x2y − xy + 2x − 1 = y2 − xy2 − 2y. Lời giải. Ta viết lại xy(x + y) − y(x + y) + 2(x + y) = 1 hay (x + y)(xy − y + 2) = 1. Đến đây xét các trường hợp hai thừa số cùng bằng 1 hoặc cùng bằng −1, giải ra được (x, y) = (0; 1),(2; −1),(−2; 1),(2; −3). Bài toán 24 (Khánh Hòa). Với mọi số nguyên dương n, chứng minh rằng q A = n2 + n2(n + 1)2 + (n + 1)2 là số nguyên nhưng không phải là số chính phương. 29 Lời giải. Ta có Tạp chí Epsilon, Số 20, 08/2021 n2 + n2(n + 1)2 + (n + 1)2 = (n2 + n + 1)2. Do đó A = n2 + n + 1, tuy nhiên dễ thấy với n > 0 thì n2 < A < (n + 1)2 nên A không thể là số chính phương. Bài toán 25 (Quảng Ngãi). 1. Cho a là số nguyên lẻ không chia hết cho 3. Chứng minh rằng a2 −20212chia hết cho 24. 2. Cho các số nguyên tố p, q sao cho p+q2là số chính phương. Chứng minh rằng p = 2q+1 và p2 + q2021 không phải là số chính phương. Lời giải. 1) Vì a không chia hết cho 3 nên a2chia 3 dư 1, mà 20212cũng thế nên 3|a2 − 20212. Lại có a lẻ nên viết a = 2k + 1 với k ∈ Z. Khi đó a2 − 1 = 4k(k + 1) chia hết cho 8. Từ đó dễ dàng có a2 − 20212chia hết cho 8 và biểu thức đã cho chia hết cho 24. 2) Đặt p + q2 = a2 với a ∈ Z+. Khi đó p = (a − q)(a + q) nên phải có a − q = 1 và a + q = p. Từ đó có ngay p = 2q + 1. Tiếp theo, giả sử phản chứng rằng p2 + q2021 = b2 với b ∈ Z+. Khi đó q2021 = (b − p)(b + p) nên tồn tại m, n ∈ Z+ sao cho m + n = 2021 và b − p = qm, b + p = qn. Suy ra 2p = qn − qm hay qn − qm = 4q + 2. Từ đó có qm(qn−m − 1) = 2(2q + 1). Dễ thấy gcd(qm, 2q + 1) = 1 nên qm|2, kéo theo q = 2, m = 1 nên 2n−1 −1 = 5, kéo theo 2n−1 = 6, vô lý. Vì thế, không thể có p2 +q2021 là số chính phương được. Bài toán 26 (Bình Định). 1. Tìm tất cả các số nguyên dương x sao cho x2 − x + 13 là số chính phương. 2. Tìm tất cả n nguyên dương để n4 + n3 + 1 là số chính phương. Lời giải. 1) Ta có (x − 1)2 < x2 − x + 13 < (x + 4)2với mọi x ∈ Z+ nên có x2 − x + 13 ∈ {x2,(x + 1)2,(x + 2)2,(x + 3)2}. Giải ra được x = 13, x = 4 là các nghiệm nguyên thỏa mãn. 2) Dễ thấy n = 1 không thỏa, còn n = 2 thì biểu thức đã cho là 25, là số chính phương. Xét n > 2 và đặt A = n4 + n3 + 1 thì ta có 4A cũng chính phương. Chú ý rằng (2n2 + n − 1)2< 4A < (2n2 + n)2. Do đó, không tồn tại số n > 2 thỏa mãn. Vì thế n = 2 là số duy nhất cần tìm. 30 Tạp chí Epsilon, Số 20, 08/2021 Bài toán 27 (Hà Tĩnh). Tìm các số nguyên m, n sao cho m(m + 1)(m + 2) = n2. Lời giải. Với m < −2 thì m(m + 1)(m + 2) < 0, không thỏa. Dễ thấy m ∈ {−2, −1, 0} thì đều có n2 = 0 tức là n = 0. Xét m > 0, ta xét các trường hợp: • Nếu m lẻ thì các số m, m + 1, m + 2 đôi một nguyên tố cùng nhau nên chúng đều phải là các số chính phương, dễ thấy không thỏa. • Nếu m chẵn thì gcd(m, m + 2) = 2 nên m + 1 lẻ và trong hai số m, m + 2 có một số chia hết cho 4 còn số kia chia 4 dư 2, kéo theo số mũ của 2 trong tích m(m + 1)(m + 2) là lẻ, không thỏa. Do đó, phương trình chỉ có ba nghiệm như trên. Bài toán 28 (Đồng Tháp). Tìm tất cả các cặp số nguyên (x, y) thỏa mãn 16x4 + y4 = 4x(x + y) − 1. Lời giải. Đặt 2x = a thì a chẵn. Ta viết lại a4 + y4 = a(a + 2y) − 1 hay a4 + y4 + y2 + 1 = (a + y)2. Nếu a = 0 thì y4 + y2 + 1 = y2, không tồn tại y. Nếu y = 0 thì a4 + 1 = a2, cũng không tồn tại a. Do đó, ta xét ay 6= 0. Tuy nhiên, nếu ay < 0 thì (a + y)2 < a2 + y2 ≤ a4 + y2 nên cũng không thỏa. Vì thế xét ay > 0. Để ý rằng nếu (a, y) thỏa mãn thì (−a, −y) cũng thế nên xét a > 0, y > 0. Ta có đánh giá quen thuộc a4 + y4 ≥(a + y)4 8 nên từ đẳng thức trên, ta có (a + y)2 >(a + y)4 8, kéo theo (a + y)2 < 8 nên (a + y)2 = 4. Mà a, y ≥ 1 nên a + y ≥ 2, từ đó phải có a = y = 1, không thỏa mãn vì a chẵn. Do đó, không tồn tại x, y thỏa mãn đề bài. Bài toán 29 (Quốc học Huế). 1. Tìm tất cả các số nguyên dương x, y thỏa mãn x2 − 2y· x − 421· 9 = 0. 31 2. Tìm tất cả các cặp số nguyên (x, y) sao cho Tạp chí Epsilon, Số 20, 08/2021 x2y − xy2 − 2x2 − 3y2 + 10xy − 16x + 21y = 100. Lời giải. 1) Coi đây là phương trình biến x, ta có ∆ = (2y)2 + 4 · 421· 9 = 4y + 9 · 422 phải là số chính phương. • Nếu y = 22 thì kiểm tra được ∆ không chính phương. • Nếu y < 22 thì ∆ = 4y(1 + 9 · 422−y) là số chính phương, kéo theo 1 + 9 · 422−ychính phương, vô lý vì 9 · 422−y đã là số chính phương. • Nếu y > 22 và viết ∆ = 422(4y−22 + 9). Từ đó đưa về xét 4y−22 + 9 là số chính phương, tức là có hai số chính phương cách nhau 9 đơn vị. Khi đó, chỉ có thể 4y−22 + 9 = 25 hay 4y−22 = 16, kéo theo y = 24. Từ đó có x = 9 · 221. 2) Ta viết lại x2(y − 2) − x(y2 − 10y + 16) − 3y2 + 21y − 30 = 70 hay (y − 2)(x + 3)(x − y + 5) = 70. Ta thấy (x + 3) − (y − 2) = x − y + 5 nên ta phân tích số 70 thành tích của ba số mà số này bằng tổng hai số kia. Dễ thấy chỉ có thể là 2 · 5 · 7 (loại −2, −5, −7 do tích âm). Từ đó thử các trường hợp và giải ra được (x, y) = (4; 4),(4; 7). Bài toán 30 (Thanh Hóa). 1. Tìm tất cả các cặp số nguyên x, y thỏa mãn x2 + 2y2 − 2xy − 2x − 4y + 6 = 0. 2. Tìm tất cả các bộ ba (x, y, z) nguyên dương sao cho √xy +√xz −√yz = y và1x+1y+1z= 1. 3. Cho số tự nhiên n ≥ 2 và số nguyên tố p thỏa mãn p − 1 chia hết cho n và n3 − 1 chia hết cho p. Chứng minh rằng n + p là số chính phương. 4. Tìm tất cả các số nguyên tố p sao cho p2−p 2 − 1 là lập phương đúng. 32 Tạp chí Epsilon, Số 20, 08/2021 Lời giải. 1) Ta viết lại x2 − 2x(y + 1) + 2y2 − 4y + 6 = 0. Coi đây là phương trình bậc hai theo biến x, ta có ∆0 = (y + 1)2 − (2y2 − 4y + 6) = −y2 + 6y − 5 ≥ 0 nên 1 ≤ y ≤ 5. Đến đây thử các trường hợp là được. 2) Ta có √x(√y +√z) = √y(√z +√y) nên x = y. Từ đó có 2x +1z = 1 nên z =x x − 2= 1 +2 x − 2. Suy ra x − 2|2 nên x = 4 hoặc x = 3. Do đó (x, y, z) = (3; 3; 3) hoặc (4; 4; 2). 3) Dễ thấy gcd(p, n) = 1. Theo đề bài thì n3 − 1 = (n − 1)(n2 + n + 1) chia hết cho p. Nếu p|n − 1 thì p ≤ n − 1 (vì n − 1 > 0), mà n|p − 1 nên p − 1 ≥ n, mâu thuẫn. Do đó ta phải có n2 + n + 1 chia hết cho p. Ngoài ra vì n|p − 1 nên p − 1 ≥ n. Từ đó suy ra n2 + n + 1 − p chia hết cho p. Mặt khác n2 + n + 1 − p cũng chia hết cho n nên ta có n2 + n + 1 − p chia hết cho np. Vì n2 + n + 1 chia hết cho p nên n2 + n + 1 − p ≥ 0, tuy nhiên nếu n2 + n + 1 − p > 0 thì ta có n2 + n + 1 − p ≥ np ≥ n(n + 1), vô lý. Vì thế n2 + n + 1 = p nên n + p = (n + 1)2là số chính phương. 4) Nếu p = 2 thì p2−p 2 − 1 = 0, thỏa mãn. Xét p > 2 thì p lẻ và x3 =p2 − p 2− 1 > 0, trong đó x ∈ Z. Ta có p2 − p − 2 = 2x3 nên p(p − 1) = 2(x + 1)(x2 − x + 1). Vì gcd(p, p − 1) = 1 nên nếu p chia hết cho x + 1 thì p = x + 1, thay vào thấy không thỏa. Nếu p − 1 chia hết cho x + 1 thì đặt p − 1 = k(x + 1) với k ∈ Z+. Khi đó k(x + 1)p = 2(x + 1)(x2 − x + 1) nên kp = 2(x2 − x + 1). Vì p lẻ nên k phải chẵn, đặt k = 2m với m ∈ Z+, thay vào có mp = x2 − x + 1. Đặt x + 1 = y ≥ 2 thì ta có x2 − x + 1 = (y − 1)2 − (y − 1) + 1 = y2 − 3y + 1 nên ta đưa về y|p − 1 và p|y2 − 3y + 1. Suy ra y2 − 3y + 1 − p chia hết cho y, p và dễ thấy gcd(y, p) = 1 nên yp|y2 − 3y + 1 − p. Rõ ràng y2 − 3y + 1 phải dương và y2 − 3y + 1 ≥ p. 33 Tạp chí Epsilon, Số 20, 08/2021 • Nếu y2 − 3y + 1 − p > 0 thì ta có y2 − 3y + 1 − p ≥ yp ≥ y(y + 1), vô lý. • Nếu p = y2 −3y +1 = x2 −x+1, khi đó p−1 = 2(x+1) nên x2 −x+1 = 2(x+1)+1, không có nghiệm nguyên. Vì thế nên p = 2 là giá trị duy nhất thỏa mãn. Bài toán 31 (PTNK TPHCM). 1. Tìm số tự nhiên n sao cho (2n + 1)3 + 1 chia hết cho 22021. 2. Cho n, p là các số tự nhiên, p nguyên tố thỏa mãn 2n+2 pvà 4n2+2n+1 pđều là các số nguyên. Chứng minh rằng hai số trên không thể đồng thời là số chính phương. Lời giải. 1) Ta có (2n + 1)3 + 1 = (2n + 2)(4n2 + 2n + 1), mà 4n2 + 2n + 1 lẻ nên cần có 22021|2n + 2 hay 22020|n + 1. Từ đó tìm được n = 22020k − 1 với k ∈ Z+. 2) Từ 4n2 + 2n + 1 chia hết cho p, ta có p lẻ nên p|n + 1. Từ đó có p|4n2 + 2n + 1 = (4n − 2)(n + 1) + 3 nên p = 3. Tiếp theo, giả sử phản chứng rằng cả 3và 4n2 + 2n + 1 2n + 2 3 đều chính phương. Suy ra tồn tại m nguyên dương để 3·4n2 + 2n + 1 2n + 2 3= m2 hay (2n + 1)3 = 9m2 − 1 = (3m − 1)(3m + 1). Vì m chẵn nên 3m − 1, 3m + 1 là hai số lẻ liên tiếp, kéo theo gcd(3m − 1, 3m + 1) = 1. Do đó cả hai số 3m − 1, 3m + 1 đều là các lập phương đúng. Tuy nhiên, không thể có hai lập phương đúng cách nhau 2 đơn vị nên điều trên là vô lý. Bài toán được giải quyết. Bài toán 32 (Chuyên KHTN Hà Nội). 1. Tìm số nguyên dương n nhỏ nhất sao cho n chia cho 7, 9, 11, 13 thì có số dư lần lượt là 3, 4, 5, 6. 2. Tìm x, y nguyên dương thỏa mãn 3x + 29 = 2y. Lời giải. 1) Theo giả thiết thì 2n + 1 chia 7, 9, 11, 13 cũng sẽ dư 7, 9, 11, 13 nên 2n + 1 chia hết cho tích 7 · 9 · 11 · 13 = 9009. Vì thế 2n + 1 ≥ 9009 nên n ≥ 4504 và thử thấy n = 4504 thỏa mãn. 34 Tạp chí Epsilon, Số 20, 08/2021 2) Nếu x = 1 thì có ngay y = 5. Xét x ≥ 2, ta có 9|2y − 29 nên 2y − 2 chia hết cho 9. Đặt y = 6k + r với k ∈ N và r ∈ {0, 1, . . . , 5} thì 2y − 2 = (26)k· 2r − 2 ≡ 2r − 2 theo mod 9. Thử trực tiếp các số r từ 0 đến 5, chỉ có r = 1 thỏa mãn. Do đó y = 6k+1 nên 2y−2 = 2(64k−1) chia hết cho 7 nên 3x + 27 chia hết cho 7. Vì thế nên 7|3x − 1. Lại đặt x = 6h + r với h ∈ N và r ∈ {0, 1, . . . , 5} tương tự trên, ta suy ra r = 0 và x = 6h. Do đó 3x − 1 = (36)h − 1 chia hết cho 33 − 1 = 26 nên 13|3x − 1. Vì thế nên 3x + 29 ≡ 4 (mod 13). Suy ra 2y ≡ 4 (mod 13), tuy nhiên, vì y = 6k + 1 nên 4 ≡ 2 · 64k ≡ 2 · (−1)k(mod 13). Dễ thấy điều trên là không thể xảy ra với mọi số tự nhiên k. Vậy nên (x, y) = (1; 5). Bài toán 33 (ĐHSP Hà Nội). 1. Cho a, b là các số hữu tỷ. Chứng minh rằng nếu a√2 + b√3 hữu tỷ thì a = b = 0. 2. Tìm tất cả số nguyên dương N sao cho N có thể biểu diễn duy nhất ở dạng x2+y nguyên dương. 3. Cho a, b, c là các số nguyên dương là lũy thừa của 2. Biết rằng phương trình ax2 − bx + c = 0 có hai nghiệm nguyên. Chứng minh rằng hai nghiệm đó phải bằng nhau. Lời giải. 1) Đặt n = a√2 + b√3 ∈ Q. Ta có n − a√2 = b√3 nên 3b = (n − a√2)2= n2 − 2an√2 + 2a ∈ Q xy+1 với x, y kéo theo a√2 ∈ Q, điều này chỉ xảy ra khi a = 0. Khi đó b√3 ∈ Q nên cũng có b = 0. 2) Với N = 1 thì ta có thể chọn (x, y) = (1; 1),(1; 2) đều có N =x2+y xy+1 , không thỏa. Xét N > 1, để ý có (x, y) = (N2, N) thỏa mãn nên luôn có ít nhất một cách biểu diễn. Ta sẽ chứng minh cách này là duy nhất. Ta có xy + 1|x2 + y nên xy + 1|y(x2 + y) − x(xy + 1) hay xy + 1|y2 − x. Ngoài ra, vì N ≥ 2 nên ta có x2 + y ≥ 2xy + 2 nên phải có x > y. Khi đó |y2 − x| < xy + 1 nên để có xy + 1|y2 − x thì phải có y2 − x = 0 hay x = y2. Thay vào N =y4+y y3+1 = y nên bộ số (x, y) xác định duy nhất. 35 Tạp chí Epsilon, Số 20, 08/2021 3) Đặt a = 2m, b = 2n, c = 2k với m, n, k ∈ N. Gọi hai nghiệm của phương trình đã cho là x1, x2 thì theo định lý Viete, ta có   x1 + x2 =ba= 2n−m x1x2 =ca= 2k−m. Từ đây ta có x1, x2 là các số nguyên dương và cũng đều là lũy thừa của 2. Đặt x1 = 2r, x2 = 2s với r, s ∈ N. Khi đó 2r + 2s = 2n−m. Nếu r 6= s, ta giả sử r > s thì có 2n−m = 2s(2r−s + 1) nên 2n−m có ước lẻ lớn hơn 1, vô lý. Vì thế nên phải có r = s và kéo theo x1 = x2. Bài toán 34 (Bắc Ninh). Tìm tất cả các số nguyên dương n có tính chất: với mỗi số nguyên dương a lẻ mà a2 ≤ n thì n chia hết cho a. Lời giải. Chú ý rằng nếu có số lẻ a ≥ 5 thì các số a, a − 2, a − 4 là đôi một nguyên tố cùng nhau. Vì thế nếu n chia hết cho các số như thế thì nó sẽ chia hết cho tích a(a − 2)(a − 4). Ta xét các trường hợp sau 1. Nếu n ≤ 8 thì số lẻ a mà a2 ≤ chỉ có 1 nên tất cả các số nguyên dương n này thỏa mãn. 2. Nếu 9 ≤ n ≤ 24 thì ta phải xét thêm số a = 3 nên chỉ có các số chia hết cho 3 mới thỏa mãn, tức là n = 9, 12, 15, 18, 21, 24. 3. Nếu 25 ≤ n ≤ 48 thì ta lại xét thêm số a = 5 nên chỉ có các số chia hết cho 15 mới thỏa mãn, tức là n = 30, n = 45. 4. Nếu 49 ≤ n ≤ 80 thì lại có thêm số a = 7 nên cần có n chia hết cho 105, rõ ràng không tồn tại số như thế. Tiếp theo, xét n ≥ 81 và đặt n = k2 + r với k ≥ 9 nguyên dương và r nguyên thỏa mãn 0 ≤ r ≤ 2k. Khi đó, nếu k lẻ thì các số k, k −2, k −4 đều là ước của n (theo nhận xét đầu tiên); còn nếu k chẵn thì k − 1, k − 3, k − 5 là ước của n. Khi đó, ta luôn có (k − 1)(k − 3)(k − 5) ≤ n ≤ k2 + 2k. Dễ thấy đánh giá này sẽ sai khi k ≥ 9. Vậy nên tất cả các số n cần tìm là n ∈ {1, 2, . . . , 8, 9, 12, 15, 18, 21, 24, 30, 45}. 36 Tạp chí Epsilon, Số 20, 08/2021 2. Các bài toán tự giải Bài toán 35 (Quảng Trị). 1. Tìm tất cả các số nguyên n để n − 1989 và n − 2022 là các số chính phương. 2. Biết rằng x2 − ax + b + 2 = 0 có hai nghiệm đều nguyên. Chứng minh rằng 2a2 + b2là hợp số. Bài toán 36 (Ninh Bình). Tìm tất cả các cặp số nguyên (x, y) sao cho 7(x + 2y)3(y − x) = 8y − 5x + 1. Bài toán 37 (Hậu Giang). Tồn tại hay không số nguyên tố p sao cho p2 + 2021 cũng là số nguyên tố? Bài toán 38 (Đồng Nai). 1. Tìm tất cả các số nguyên x, y thỏa mãn 5x2 + 3y2 + 4xy − 2x + 8y + 8 ≤ 0. 2. Hỏi trong 2021 số nguyên dương đầu tiên, có bao nhiêu số không chia hết cho 7 và không chia hết cho 11? Bài toán 39 (Ninh Thuận). Tìm tất cả các số nguyên x, y thỏa mãn y2 + 3y = x4 + x2 + 18. Bài toán 40 (Lai Châu). Chứng minh rằng với mọi số tự nhiên n thì 2004|2005n + 60n − 1897n − 168n. Bài toán 41 (Gia Lai). Tìm nghiệm nguyên của phương trình x2 − 2y(x − y) = 2(x − 1). Bài toán 42 (Kon Tum). Tìm tất cả các cặp số nguyên (x, y) thỏa mãn x3y − x3 − 1 = 2x2 + 2x + y. Bài toán 43 (Bắc Ninh). Tìm tất cả các số nguyên (x, y) thỏa mãn x(1 + x + x2) = 4y(y − 1). Bài toán 44 (Hải Dương). Tìm tất cả các bộ số nguyên (x, y, z) thỏa mãn x2 + 4y2 + z2 + 2(xz + 2x + 2z) = 396 và x2 + y2 = 3z. Bài toán 45 (Hải Phòng). Tìm các cặp số nguyên dương (x, y) thỏa mãn y4 + 2y2 − 3 = x2 − 3x. 37 Tạp chí Epsilon, Số 20, 08/2021 Bài toán 46 (Kon Tum). Hỏi có bao nhiêu số tự nhiên có 6 chữ số dạng 357abc sao cho số này chia hết cho 3, 5 và 7. Bài toán 47 (Hòa Bình). Hỏi có bao nhiêu số tự nhiên n ≤ 2021 mà n3 + 2021 chia hết cho 6? Bài toán 48 (Yên Bái). 1. Chứng minh rằng nếu n là số nguyên thì n2 + 2022 không là lũy thừa của 3. 2. Tìm tất cả các số nguyên tố p sao cho p2 + 3p + 2021 cũng là số nguyên tố. Bài toán 49 (Quảng Bình). Tìm tất cả các số nguyên dương n sao cho n2 − 2n − 7 và n2 − 2n + 12 đều là lập phương của hai số nguyên dương nào đó. Bài toán 50 (Thái Nguyên). 1. Tìm n nguyên dương để A = 4n3 + 2n2 − 7n − 5 là một số chính phương. 2. Tìm các số nguyên tố p, q sao cho phương trình x2 − px + q = 0 có các nghiệm đều nguyên. Bài toán 51 (Hải Dương). Tìm tất cả các số tự nhiên abcd thỏa mãn đồng thời: abcd chia hết cho 3 và abc − bda = 650. Bài toán 52 (Hưng Yên). Tìm nghiệm nguyên của phương trình 2x2 + 5y2 + 4x = 4y + 4xy + 19. Bài toán 53 (ĐH Khoa học Huế). Tìm tất cả các số tự nhiên a > 1, b > 1 để ab − 1 chia hết cho (a − 1)(b − 1). Bài toán 54 (Cà Mau). Tất cả học sinh lớp 9 của trường THCS Tân Tiến tham gia xếp hàng đồng diễn thể dục, mỗi hàng không quá 25 học sinh. Nếu xếp mỗi hàng 16 học sinh thì còn thừa một em, nếu bớt đi một hàng thì có thể chia đều tất cả học sinh vào các hàng còn lại sao cho học sinh mỗi hàng bằng nhau. Tính số lượng học sinh. Bài toán 55 (Quảng Ninh). Cho hình lăng trụ đứng, đáy là tam giác vuông, chiều cao bằng 6. Số đo ba cạnh của đáy là nguyên. Số đo diện tích toàn phần của lăng trụ bằng số đo thể tích của nó. Tính số đo ba cạnh của đáy lăng trụ. Bài toán 56 (Đăk Nông). Tìm tất cả các số nguyên (x, y) thỏa mãn x2 + 3y2 + 4xy − 2x − 4y − 2 = 0. 38 Tạp chí Epsilon, Số 20, 08/2021 BỔ ĐỀ HOÁN VỊ (2) Tạ Hồng Quảng, Nguyễn Văn Huyện Bất đẳng thức hoán vị luôn là một trong những chủ đề khó và thú vị, đặc biệt là các bài toán có dấu bằng không bình thường, lệch tâm, lệch biên, lượng giác,... chúng rất khó để tìm các đánh giá trung gian mà vẫn đảm bảo dầu bằng. Thông thường những bài toán dạng này chúng ta sử dụng kỹ thuật pqr để đưa về khảo sát cực trị của hàm số theo q; theo r hoặc phân tích về dạng tổng các bình phương, ... Trong bài viết này, tác giả sẽ giới thiệu cách xử lý lớp bất đẳng thức hoán vị có dạng ak bCbkcCcka; ab C bc C ca; a C b C c > 0; k D 1; 2; 3: f Trường hợp k D 1 đã được giới thiệu trong [1], bài viết sẽ giới thiệu sơ về trường hợp k D 2 và tập trung khai thác k D 3: Với a; b; c là ba số thực dương, ký hiệu p D a C b C c; q D ab C bc C ca và r D abc: 1. Trường hợp k D 2 Bổ đề 1. (Võ Quốc Bá Cẩn) Giả sử a C b C c D 1 và đặt q D 1t2 minh rằng 3; với 0 6 t < 1: Chứng bCb2cCc2a>1 C 2t C 5t 2 C 4t 3 3t 4 a2 Lời giải. Đặt ta cần chứng minh .2t C 1/.1 t2/: (1) M D1 C 2t C 5t 2 C 4t 3 3t 4 .2t C 1/.1 t2/; a2 bCb2cCc2a> M: Chuyển sang pqr 1 như sau q.p2 2q/ pr C p.a b/.b c/.c a/ > 2M r; 1Xem thêm tại: https://wp.me/p7ChCZ-ht 39 hay là Ta chứng minh Thật vậy, ta có Tạp chí Epsilon, Số 20, 08/2021 q.1 2q/ .2M C 1/r > .a b/.b c/.a c/: q.1 2q/ .2M C 1/r > 0: (2) q.1 2q/ .2M C 1/r D.2t 2 C 1/.1 t2/ 9 Mặt khác, từ 3.1 C 2t C 3t 2 C 2t 3 2t 4/r .1 C 2t /.1 t2/: 0 6 27.a b/2.b c/2.c a/2 D 4.p2 3q/3 .2p3 9pq C 27r/2; ta được 0 < r 6 r0 Dp.9q 2p2/ C 2.p2 3q/pp2 3q 27D.1 C 2t /.1 t /2 27; (3) và.2t 2 C 1/.1 t2/ 9 3.1 C 2t C 3t 2 C 2t 3 2t 4/ .1 C 2t /.1 t2/ .1 C 2t /.1 t /2 27 D2t 3.2t C 1/.1 t / 9.t C 1/ > 0: Cho nên (2) đúng. Quay trở lại bài toán, ta cần chứng minh q.1 2q/ .2M C 1/r > .a b/.b c/.a c/; hay là q.1 2q/ .2M C 1/r >p.a b/2.b c/2.c a/2: Bình phương hai vế và nhóm lại theo r như sau Œq.1 2q/ .2M C 1/r2 > q2.p2 4q/ 2p.2p2 9q/r 27r2; hoặc .M2 C M C 7/r2 C Œ.2M C 1/q2 .M C 5/q C 1r C q4 > 0: (4) Lưu ý rằng M2 C M C 7 D9.t 4 t3 C 2t C 1/2 .2t C 1/2.t 1/2.t C 1/2; .2M C 1/q2 .M C 5/q C 1 D 2.1 t2/.t 4 t3 C 2t C 1/ 3.2t C 1/ ; nên (4) tương đương với .2t C 1/2.t 1/2.t C 1/2 r2 2.1 t2/.t 4 t3 C 2t C 1/ 9.t 4 t3 C 2t C 1/2 3.2t C 1/ C.1 t2/4 81> 0; thu gọn thành 1 81 27.t 4 t3 C 2t C 1/ .2t C 1/.1 t2/ r .1 t2/2 40 2 > 0: Tạp chí Epsilon, Số 20, 08/2021 Đẳng thức xảy ra khi và chỉ khi a; b; c là ba nghiệm của hệ 8< : Chứng minh hoàn tất. .a b/.b c/.c a/ 6 0 x3 x2 C1 t2 3 x .2t C 1/.1 t2/3 27.t 4 t3 C 2t C 1/ D 0(5) Nhận xét. Nếu a C b C c D 1; từ bất đẳng thức quen thuộc .a C b C c/2 > 3.ab C bc C ca/ ta được q 613: Do đó ta luôn có thể đặt q D 1t2 có trường hợp 0 6 t < 1: 2. Các bài toán ứng dụng 3 613; với 0 6 t 6 1; nhưng vì q > 0 nên chỉ Bài toán 1. (Phạm Hữu Đức) Chứng minh rằng a2 bCb2cCc2aC a C b C c >6.a2 C b2 C c2/ a C b C c: Lời giải. Chuẩn hóa a C b C c D 1 và đặt q D 1t2 3với 0 6 t < 1; bất đẳng thức trở thành a2 bCb2cCc2a> 5 12q; hay là a2 bCb2cCc2a> 4t 2 C 1: Sử dụng (1), ta đưa bài toán về chứng minh 1 C 2t C 5t 2 C 4t 3 3t 4 .2t C 1/.1 t2/> 4t 2 C 1; thu gọn thành t2Œ8t 3 C 1 C .t 1/2 .2t C 1/.1 t2/> 0: Hiển nhiên đúng, nên bài toán được chứng minh. Đẳng thức xảy ra khi a D b D c: Bài toán 2. (Michael Rozenberg) Chứng minh rằng a2 bCb2cCc2a>37.a2 C b2 C c2/ 19.ab C ac C bc/ 6.a C b C c/ : Lời giải. Chuẩn hóa a C b C c D 1; và viết bất đẳng thức lại như sau a2 bCb2cCc2a>316t2 C 1: 41 Tạp chí Epsilon, Số 20, 08/2021 Sử dụng (1), ta đưa bài toán về chứng minh 1 C 2t C 5t 2 C 4t 3 3t 4 thu gọn thành Lưu ý rằng .2t C 1/.1 t2/>316t2 C 1; t2.62t 3 C 13t 2 26t C 5/ 6.2t C 1/.1 t2/> 0: 62t 3 C 13t 2 26t C 5 D3150t .10t 3/2 C2515 t 1579 2 5020 C16759 502000> 0: Đẳng thức xảy ra khi và chỉ khi a D b D c: Bài toán được chứng minh. Bài toán 3. (Tạ Hồng Quảng) Với a C b C c D 1; chứng minh rằng bCb2cCc2a>804q2 633q C 127 a2 Lời giải. Bất đẳng thức tương đương với a2 16.1 2q/ : bCb2cCc2a>268t 4 C 97t 2 C 16 16.2t 2 C 1/ : Sử dụng (1), ta cần chứng minh 1 C 2t C 5t 2 C 4t 3 3t 4 .2t C 1/.1 t2/>268t 4 C 97t 2 C 16 16.2t 2 C 1/ ; thu gọn thành t2.1 2t /2.134t 3 C 177t 2 C 90t C 31/ 16.2t C 1/.2t 2 C 1/.1 t2/> 0: Đẳng thức xảy ra khi và chỉ khi t D 0 hoặc t D 12; tức a D b D c hoặc a; b; c là nghiệm của 8< .a b/.b c/.c a/ 6 0 x3 x2 Cx4162D 0 D 0 : Chứng minh hoàn tất. 3. Bài tập rèn luyện Bài tập 1. (Tạ Hồng Quảng) Với a C b C c D 1; chứng minh rằng a2 bCb2cCc2aC155.ab C bc C ca/ 32.a2 C b2 C c2/>307 64: a2 bCb2cCc2aC2945 32.ab C bc C ca/ >3667 128: 42 Tạp chí Epsilon, Số 20, 08/2021 Bài tập 2. (Michael Rozenberg) Tìm hằng số dương k lớn nhất để bất đẳng thức luôn đúng a2 bCb2cCc2a>k.a2 C b2 C c2/ .k 3/.ab C bc C ca/ a C b C c: Bài tập 3. Chứng minh rằng bCb2cCc2a>r61.a2 C b2 C c2/ 46.ab C bc C ca/ a2 Bài tập 4. (Nguyễn Văn Thạch) Chứng minh rằng s a2 bCb2cCc2a> 3 Bài tập 5. (Võ Quốc Bá Cẩn) Chứng minh rằng 5: a4 C b4 C c4 a2 C b2 C c2: bCb2cCc2a> 36ra6 C b6 C c6 a2 4. Trường hợp k D 3 3: Bổ đề 2. (Tạ Hồng Quảng) Giả sử a Cb Cc D 1 và đặt q D 1 t2C3với t > 0: Chứng minh rằng a3 bCb3cCc3a>t5 C t4 C 2 t 3 C 4 t 2 C t C 1 .t C 1/.t 2 C 3/ : (6) Lời giải. Đặt ta chứng minh Chuyển sang pqr như sau M Dt5 C t4 C 2t 3 C 4 t 2 C t C 1 .t C 1/.t 2 C 3/ ; a3 bCb3cCc3a> M: pq.p2 3q/ .p2 5q/r C .p2 q/.a b/.b c/.c a/ > 2M r; hay là pq.p2 3q/ C .5q p2 2M /r > .p2 q/.a b/.b c/.a c/; hoặc q.1 3q/ C .5q 1 2M /r > .1 q/.a b/.b c/.a c/: Ta chứng minh q.1 3q/ C .5q 1 2M /r > 0: (7) 43 Tạp chí Epsilon, Số 20, 08/2021 t2C3; ta có r 6q2 Thật vậy, với q D 1 3p D 1 3.t2C3/2; và q.1 3q/ C .5q 1 2M /r Dt2 .t 2 C 3/2 t2.2t 3 C 2t 2 C 5t C 9/ r .t C 1/.t 2 C 3/ ; đồng thời Dt2 t2 C 3 1 t2 C 3 .2t 3 C 2t 2 C 5t C 9/ r t C 1 2t 3 C 2t 2 C 5t C 9 1 3.t 2 C 3/2Dt .t 2 C t C 4/ t2 C 3 t C 1 1 3.t C 1/.t 2 C 3/2> 0: Cho nên bất đẳng thức (7) đúng. Quay trở lại bài toán, ta cần chứng minh q.1 3q/ C .5q 1 2M /r > .1 q/.a b/.b c/.a c/; hay là q.1 3q/ C .5q 1 2M /r > .1 q/p.a b/2.b c/2.c a/2: Bình phương hai vế và nhóm lại theo r như sau q.1 3q/ C .5q 1 2M /r 2> .1 q/2.a b/2.b c/2.c a/2; q.1 3q/ C .5q 1 2M /r 2> .1 q/2 q2.1 4q/ C 2.9q 2/r 27r2 ; tương đương với 13q2 .5M C 16/q CM2 CM C 7 r2 12q3 .3M C 14/q2 C .M C 7/q 1 r C q5 > 0: Thay giá trị của q và M theo t; ta được 13q2 .5M C 16/q C M2 C M C 7 D.t 4 C t3 C t2 C 3t C 3/2 .t C 1/2.t 2 C 3/ ; 12q3 .3M C 14/q2 C .M C 7/q 1 D2.t 4 C t3 C t2 C 3t C 3/ .t C 1/.t 2 C 3/3: Do đó bất đẳng thức cần chứng minh trở thành .t 4 C t3 C t2 C 3t C 3/2 .t C 1/2.t 2 C 3/ r2 2.t 4 C t3 C t2 C 3t C 3/ .t C 1/.t 2 C 3/3 r C1 .t 2 C 3/5> 0; thu gọn lại dưới dạng 1 t2 C 3 t4 C t3 C t2 C 3t C 3 t C 1 r 1 .t 2 C 3/2 2 > 0: Đẳng thức xảy ra khi và chỉ khi a; b; c là ba nghiệm của 8< .a b/.b c/.c a/ 6 0 x3 x2 Cx : t2 C 3 .t 4 C t3 C t2 C 3t C 3/.t 2 C 3/2D 0(8) t C 1 Chứng minh hoàn tất. Nhận xét. Tương tự như trên, với a C b C c D 1 ta có thể đặt q D 1 t2C3 613; với t > 0: 44 Tạp chí Epsilon, Số 20, 08/2021 5. Các bài toán ứng dụng Bài toán 4. (Michael Rozenberg) Chứng minh rằng a3 bCb3cCc3aC 2.ab C bc C ca/ > 3.a2 C b2 C c2/: (9) Lời giải. Chuẩn hoá a C b C c D 1 và đặt q D 1 t2C3với t > 0; khi đó 3.a2 C b2 C c2/ 2.ab C bc C ca/ D 3p2 8q D 3 8q D 3 8 t2 C 3: Sử dụng (6), ta đưa bài toán về chứng minh t5 C t4 C 2t 3 C 4t 2 C t C 1 .t C 1/.t 2 C 3/ > 3 8 t2 C 3; thu gọn thành t2.t 3 C t2 t C 1/ .t C 1/.t 2 C 3/ > 0: (10) Dễ thấy (10) đúng nên ta có điều phải chứng minh. Đẳng thức xảy ra khi a D b D c: Bài toán 5. (Sladjan Stankovik) Chứng minh rằng .a b/2 a b 2 3 C .b c/2 b c 2 3 C .c a/2 c a 2 3 > 0: Lời giải. Bất đẳng thức tương đương với a3 bCb3cCc3a C 7.ab C bc C ca/ > 10.a2 C b2 C c2/: 3 Chuẩn hóa a C b C c D 1 và viết lại dưới dạng a3 bCb3cCc3a C27 3 t2 C 3> 10: Sử dụng (6), ta đưa bài toán về chứng minh 3.t 5 C t4 C 2t 3 C 4t 2 C t C 1/ .t C 1/.t 2 C 3/ C27 t2 C 3> 10; thu gọn lại thành Bài toán được chứng minh. t2.3t 3 C 3t 2 4t C 2/ .t C 1/.t 2 C 3/ > 0: 45 Tạp chí Epsilon, Số 20, 08/2021 Bài toán 6. (Trần Quốc Anh) Chứng minh rằng bất đẳng thức dưới đây luôn đúng a3 bCb3cCc3a> .k C 1/.a2 C b2 C c2/ k.ab C bc C ca/; với mọi 0 6 k 6 k0; với k0 là nghiệm dương của phương trình k3 5k2 C 26k 51 D 0: Lời giải. Chuẩn hoá a C b C c D 1; ta có .k C 1/.a2 C b2 C c2/ k.ab C bc C ca/ D k C 1 3k C 2 t2 C 3: Sử dụng (6), ta đưa bài toán về chứng minh t5 C t4 C 2t 3 C 4t 2 C t C 1 .t C 1/.t 2 C 3/ > k C 1 3k C 2 t2 C 3; hay là Ta chứng minh Thật vậy t2 t3 C t2 C .1 k/t C 3 k .t C 1/.t 2 C 3/ > 0: f .k/ D t3 C t2 C .1 k/t C 3 k > 0: Nếu 0 6 k < 23; thì hiển nhiên f .k/ > 0: Nếu 23 6 k 6 k0; đặt m2 D 3k 2; ta có m2 C 2 3C7 m2 f .k/ D f D t3 C t2 .m2 1/t 3 3 27C.3t C 2m C 1/.m 3t 1/2 D2.31 3m2 m3/ Lưu ý rằng k 6 k0 < 3; nên 27: 31 3m2 m3 D 37 9k p.3k 2/3 D 27.k3 5k2 C 26k 51/ 37 9k Cp.3k 2/3D 0: Do đó f .k/ D.3t C 2m C 1/.m 3t 1/2 27> 0: Đẳng thức xảy ra khi và chỉ khi t D 0 hoặc t D Với t D 0; ta được a D b D c: p3k21 3 Với t D p3k21 3; phần này bạn đọc có thể dựa vào (8) để tìm a; b; c: 46 Tạp chí Epsilon, Số 20, 08/2021 Chứng minh hoàn tất. Nhận xét. Giá trị cụ thể của k0 là p31828 C 372p93 p31828 C 372p93C53 2:581 k0 D 6 106 3 Trường hợp k D 2 ta được bất đẳng thức (9). Bài toán 7. (Tạ Hồng Quảng, Nguyễn Văn Huyện) Với a C b C c D 1; chứng minh rằng a3 144>6518C.4 13q/2 bCb3cCc3aC1417.ab C bc C ca/ Lời giải. Ta viết bất đẳng thức lại như sau 1 C 141q: a3 144.t 2 C 3/ >6518C.2t 1/2.2t C 1/2 bCb3cCc3aC1417 Sử dụng (6), ta cần chứng minh .t 2 C 144/.t 2 C 3/: t5 C t4 C 2t 3 C 4t 2 C t C 1 144.t 2 C 3/ >6518C.2t 1/2.2t C 1/2 .t 2 C 144/.t 2 C 3/; thu gọn thành .t C 1/.t 2 C 3/ C1417 hay là .36t 3 C 72t 2 C 5t C 1/.2t 1/2 144.t C 1/.t 2 C 3/ >.2t 1/2.2t C 1/2 .t 2 C 144/.t 2 C 3/; t2.2t 1/2.36t 3 C 72t 2 C 4613t C 9217/ 144.t C 1/.t 2 C 3/.t 2 C 144/ > 0: Đẳng thức xảy ra khi và chỉ khi t D 0 hoặc t D 12; tức a D b D c hoặc a; b; c là nghiệm của 8< .a b/.b c/.c a/ 6 0 x3 x2 C413x 384 : 13351D 0 Chứng minh hoàn tất. Bài toán 8. (Nguyễn Văn Huyện) Với a C b C c D 1; chứng minh rằng bCb3cCc3a C 18.ab C bc C ca/ > 19.a2 C b2 C c2/ C3.1 4q/2 4 a3 3 8q: Lời giải. Viết bất đẳng thức lại như sau a3 bCb3cCc3a C 56q 19 >3.1 4q/2 4 3 8q; 47 hay là bCb3cCc3a 19t 2 C 1 Tạp chí Epsilon, Số 20, 08/2021 4 a3 t2 C 3>3.t 1/2.t C 1/2 .3t 2 C 1/.t 2 C 3/: Áp dụng (6), ta cần chứng minh 4.t 5 C t4 C 2t 3 C 4t 2 C t C 1/ .t C 1/.t 2 C 3/ tương đương với 19t 2 C 1 t2 C 3>3.t 1/2.t C 1/2 .3t 2 C 1/.t 2 C 3/; thu gọn thành .4t 3 C 12t 2 C 9t C 3/.t 1/2 .t C 1/.t 2 C 3/ >3.t 1/2.t C 1/2 .3t 2 C 1/.t 2 C 3/; 4t 2.t 1/2.3t 3 C 9t 2 C 7t C 3/ .t C 1/.t 2 C 3/.3t 2 C 1/ > 0: Đẳng thức xảy ra khi và chỉ khi t D 0 hoặc t D 1; tức a D b D c hoặc a; b; c là nghiệm của 8< (11) : Chứng minh hoàn tất. x3 x2 Cx4172D 0 .a b/.b c/.c a/ 6 0 Bài toán 9. (Tạ Hồng Quảng) Với a C b C c D 2; chứng minh rằng a3 bCb3cCc3a>.q C 4/.3 2q/ q: Lời giải. Viết lại bài toán dưới dạng điều kiện a C b C c D 1 a3 bCb3cCc3a>.q C 1/.3 8q/ 4q; hay là a3 bCb3cCc3a>.t 2 C 4/.3t 2 C 1/ 4.t 2 C 3/ : Sử dụng (6), ta cần chứng minh t5 C t4 C 2t 3 C 4t 2 C t C 1 .t C 1/.t 2 C 3/ >.t 2 C 4/.3t 2 C 1/ 4.t 2 C 3/ ; thu gọn thành t2.t 1/2.t C 3/ 4.t C 1/.t 2 C 3/ > 0: Đẳng thức xảy ra khi và chỉ khi t D 0 hoặc t D 1: 48 Tạp chí Epsilon, Số 20, 08/2021 Bài toán 10. (Nguyễn Văn Huyện) Với a C b C c D 1; chứng minh rằng a3 bCb3cCc3a C 221.ab C bc C ca/ > 94.a2 C b2 C c2/ C17.1 7q/2 9 17 49q: Lời giải. Viết bất đẳng thức lại như sau a3 bCb3cCc3a C409 9 Sử dụng (6), ta cần chứng minh t2 C 3> 94 C17.t 2/2.t C 2/2 .17t 2 C 2/.t 2 C 3/: 9.t 5 C t4 C 2t 3 C 4t 2 C t C 1/ t2 C 3> 94 C17.t 2/2.t C 2/2 .17t 2 C 2/.t 2 C 3/; thu gọn thành .t C 1/.t 2 C 3/ C409 .t C 1/.t 2 C 3/ >17.t 2/2.t C 2/2 .9t 3 C 45t 2 C 68t C 34/.t 2/2 .17t 2 C 2/.t 2 C 3/; hay là t2.t 2/2.153t 3 C 765t 2 C 1157t C 583/ .t C 1/.t 2 C 3/.17t 2 C 2/ > 0: Đẳng thức xảy ra khi t D 0 hoặc t D 2; tức a D b D c hoặc a; b; c là nghiệm của 8< x3 x2 Cx73 1813D 0 : .a b/.b c/.c a/ 6 0 Chứng minh hoàn tất. Bài toán 11. (Nguyễn Văn Huyện) Chứng minh rằng a3 bCb3cCc3aC 39.ab C bc C ca/ > 13.a2 C b2 C c2/: Lời giải. Chuẩn hóa a C b C c D 1; ta viết bất đẳng thức lại như sau a3 bCb3cCc3aC65 t2 C 3> 13: Áp dụng (6), ta cần chứng minh t5 C t4 C 2t 3 C 4t 2 C t C 1 .t C 1/.t 2 C 3/ C65 t2 C 3> 13; thu gọn thành .t C 3/.t 2 t 3/2 .t C 1/.t 2 C 3/ > 0: 49 Đẳng thức xảy ra khi và chỉ khi t D 1Cp13 Tạp chí Epsilon, Số 20, 08/2021 2; tức a; b; c là nghiệm của x3 x2 C13 p13 78x C3 p13 8< 702D 0 : .a b/.b c/.c a/ 6 0 Chứng minh hoàn tất. Bài toán 12. (Tạ Hồng Quảng) Với a C b C c D 1; chứng minh rằng bCb3cCc3a C 56q > 19 C.292 795q/ .1 4q/2 4 a3 27q .3 8q/: Lời giải. Ta viết bất đẳng thức lại như sau bCb3cCc3a C56 4 a3 t2 C 3 19 >.292t 2 C 81/.t 1/2.t C 1/2 27.3t 2 C 1/.t 2 C 3/ : Áp dụng (6), ta cần chứng minh 4.t 5 C t4 C 2t 3 C 4t 2 C t C 1/ t2 C 3 19 >.292t 2 C 81/.t 1/2.t C 1/2 27.3t 2 C 1/.t 2 C 3/ ; hay là .t C 1/.t 2 C 3/ C56 .4t 3 C 12t 2 C 9t C 3/.t 1/2 .t C 1/.t 2 C 3/ >.292t 2 C 81/.t 1/2.t C 1/2 27.3t 2 C 1/.t 2 C 3/ ; thu gọn thành 8t 2.t 1/2.2t 1/2.t C 4/ 27.t C 1/.t 2 C 3/.3t 2 C 1/ > 0: Đẳng thức xảy ra khi và chỉ khi t D 0; t D 12hoặc t D 1: 6. Bài tập rèn luyện Bài tập 6. (Tạ Hồng Quảng, Nguyễn Văn Huyện) Với a C b C c D 1; chứng minh rằng bCb3cCc3aC28q2 a3 3.1 2q/ >2912C37.1 4q/2 37 99q: Bài tập 7. (Tạ Hồng Quảng) Chứng minh rằng a3 bCb3cCc3a 2C452.ab C bc C ca/2 >958.a2 C b2 C c2/2 Bài tập 8. (Tạ Hồng Quảng) Chứng minh rằng a3 bCb3cCc3aC28.ab C bc C ca/2 .a C b C c/2> 3.a C b C c/2: 50 Tạp chí Epsilon, Số 20, 08/2021 Bài tập 9. (Tạ Hồng Quảng) Chứng minh rằng a3 bCb3cCc3aC12.ab C bc C ca/2 .a C b C c/2> 4.a2 C b2 C c2/: Bài tập 10. (Tạ Hồng Quảng) Với a C b C c D 1; chứng minh rằng a3 bCb3cCc3aC2863.ab C bc C ca/2 18>907 126: Kết thúc bài viết, xin chân thành cám ơn các độc giả đã dành thời gian theo dõi, hy vọng trong số các bạn sẽ có người tìm được giải pháp tương tự cho k > 3: Tài liệu tham khảo [1] https://nguyenhuyenag.files.wordpress.com/2020/07/ bo-de-hoan-vi.pdf [2] https://artofproblemsolving.com/community/c6t243f6h432676 [3] Chuyên đề Bất Đẳng Thức Hiện Đại, Võ Quốc Bá Cẩn. 51 Tạp chí Epsilon, Số 20, 08/2021 CÁC BÀI TOÁN TẬP HỢP CÓ NHIỀU ĐIỀU KIỆN Võ Thịnh Phát Trường THPT chuyên Thoại Ngọc Hầu, An Giang Khóa 2019 2022 Qua bài viết này, tác giả muốn giới thiệu một lớp các bài toán có chứa một hay nhiều tập hợp có nhiều điều kiện ràng buộc với nhau. Các ràng buộc này có thể là giữa hai tập hợp với nhau hoặc cũng có thể là giữa các phần tử trong cùng một tập hợp. Khi đó, ta cần những nhận xét hay nhìn nhận tinh tế để sử dụng khéo léo các điều kiện đó. 1. Kiến thức chuẩn bị 1.1. Nguyên lý quy nạp Ta thường dự đoán các tính chất trong tập hợp, hoặc một nhóm các phần tử có công thức chung thuộc một tập hợp nào đó. Khi đó, công cụ quy nạp hỗ trợ rất đắc lực trong việc chứng minh dự đoán. Để chứng minh mệnh đề P .n/ đúng với mọi số tự nhiên n, ta thực hiện hai bước : Bước cở sở : Kiểm tra mệnh đề đúng với P .0/ (có khi là P .1/) Bước quy nạp : Giả sử mệnh đề đúng đến n D k, tức là P .k/ đúng. Dựa vào đó, ta sẽ chứng minh P .k C 1/ cũng đúng. 1.2. Phương pháp phản chứng Cơ sở của phương pháp phản chứng là : .A ) B/ , .B ) A/ với A; B là hai khẳng định. Vậy để chứng minh mệnh đề A ) B, ta sẽ giả sử điều ngược lại với B, từ đó bằng các lập luận, ta sẽ chỉ ra điều vô lý. Cụ thể hơn, trong các bài toán tập hợp, để chứng minh a 2 A, ta có thể giả sử a … A, từ đó chỉ ra điều vô lý. 52 Tạp chí Epsilon, Số 20, 08/2021 1.3. Nguyên lý cực hạn Trong tập hợp hữu hạn phần tử luôn luôn tồn tại phần tử lớn nhất và phần tử nhỏ nhất. Hệ quả : Mọi tập con của tập hợp số tự nhiên luôn chứa phần tử nhỏ nhất. Đôi lúc, ta sẽ gọi phần tử nhỏ nhất này ra và làm việc với nó để thu thêm các dữ kiện khác. 1.4. Nguyên lý bao hàm và loại trừ Cho n tập hợp A1; A2; :::; An. Khi đó jA1 [ A2 [ [ Anj D Xn iD1 jAij X i 1 thì : xnC1 D 1xn nếu xn ¤ 0, ngược lại xnC1 D 0 nếu xn D 0. Ta thấy .xn/ là dãy hữu tỉ dương, và nếu tồn tại t là số nhỏ nhất sao cho xt D 0 thì xn D 0 với mọi n > t, đồng thời 1 xt1 D xt D 0, nên1 xt12 N A, suy ra xt1 2 A, từ đó xt2 D k C xt1 2 A; k 2 N nào đó. Tiếp tục thực hiện tương tự, ta thu được x1 Dpq2 A. Ngược lại, nếu không tồn tại số t có tính chất vừa xét thì xn 2 .0; 1/ với mọi n. Khi đó, qn là mẫu số của xn khi viết ở dạng phân số tối giản thì .qn/ là dãy nguyên dương, vô hạn và giảm ngặt, đây là điều vô lý. Từ các lập luận trên, ta suy ra A D Q . Bài 3. Cho tập hợp X D f1; 2; :::; 396g. Gọi S1; S2; :::; Sk là k tập con khác nhau của X thỏa mãn đồng thời hai điều kiện sau : (i) jS1j D jS2j D ::: D jSkj D 198 54 Tạp chí Epsilon, Số 20, 08/2021 (ii) jSi \ Sj j 6 99; 8i; j 2 N ; 1 6 i < j 6 k: Chứng minh rằng k 6 650. Lời giải. Từ điều kiện .i i / ta thấy mỗi bộ 100 phần tử chỉ có thể được chứa tối đa trong 1 tập hợp. Ta đếm các bộ fx1; x2; :::; x100; Mg, trong đó xi 2 X với mọi i và M là một trong các tập Si, M chứa x1; x2; :::; x100. Số cách chọn M là k. Số cách chọn 100 phần tử trong M là C100 198 . Số cách chọn x1; x2; :::; x100 từ X là C100 396 . Với mỗi bộ 100 phần tử như vậy, có tối đa 1 tập Sithỏa mãn Si chứa x1; x2; :::; x100. Do đó, ta có : k C100 198 6 C100 396 , k 6 650. Bài 4. Cho tập hợp A thỏa mãn Z A R và thỏa mãn đồng thời các điều kiện : 1. Nếu x; y 2 A thì xy; x C y 2 A 2. pn Cpn C 1 2 A với mọi số tự nhiên n. Chứng minh1 .pn Cpn C 1/20212 A: Lời giải. Ta sẽ chứng minh bằng quy nạp pn 2 A với mọi số tự nhiên n. Trước hết p0 D 0 2 A. Giả sử khẳng định đúng đến n > 0. Ta có : pn Cpn C 1 2 A và 1 2 A; pn 2 A, nên : pn 2 A. Suy ra : .pn Cpn C 1/ pn Dpn C 1 2 A. Theo quy nạp thì khẳng định được chứng minh. Vì thế với mọi số tự nhiên n : pn C 1 C .pn/ D1 pn Cpn C 12 A Theo .i / thì x 2 A thì x2021 cũng thuộc A. Nên ta có điều phải chứng minh. Bài 5 (Hà Nam Ninh 2021). Cho tập hợp S D f1; 2; 3; :::; 2020g. Ta chia S thành hai tập con khác rỗng rời nhau A và B thỏa mãn các điều kiện sau : (i) 13 2 A (ii) Nếu a 2 A; b 2 B; a C b 2 S thì a C b 2 B. (iii) Nếu a 2 A; b 2 B; ab 2 S thì ab 2 A. 55 a) Chứng minh rằng 1 2 B b) Tìm số phần tử của tập hợp A. Lời giải. a) Nhận xét : Nếu m > n đều thuộc A thì m n thuộc A. Nếu mn; m 2 B thì n 2 B. Do đó, vì B khác rỗng nên 1 2 B. Tạp chí Epsilon, Số 20, 08/2021 b) Từ đó, ta suy ra : 14; 27; :::; 13k C 1 2 B .k D 1; 2; :::; 155/ Nếu 2 (hoặc 3, 4, 6) thuộc A thì 1 thuộc A. Do đó 2, 3, 4, 6 thuộc B. Ta lại có : 2:2:2I 7 D142I 8 D405I 10 D40 2:2I 11 D66 2:3I 12 D144 2:3 đều thuộc B. 5 D40 Vậy 1; 2; 3; :::; 12 thuộc B. Suy ra : 14; 15; :::; 25 thuộc B, từ đó : 27; 28; :::; 38 thuộc B. Do đó : nếu n không chia hết cho 13 thì n thuôc B. Mà 13 2 A nên 13:2; 13:3; :::; 13:14 đều thuộc A. Suy ra : 132 D 13:14 13 2 A. Vậy nếu n chia hết cho 13 thì n thuộc A. Cho nên jAj D 155. Bài 6. Cho 2021 số thực dương a1; a2; :::; a2021 và F là một tập con của R thỏa mãn đồng thời các điều kiện sau đây : (i) a2k2 F với mỗi chỉ số k, đồng thời a1 C a2 C ::: C a2021 2 F (ii) Nếu x; y 2 F; y ¤ 0 thì x y 2 F vàxy2 F Chứng minh rằng ak 2 F với mỗi chỉ số k. Lời giải. Để ý rằng 0 D a21 a21 2 F; 1 Da21 a212 F , từ đó cứ x 2 F thì sẽ phải có được x D 0 x 2 F và với x ¤ 0; x 2 F thì1x2 F . Do đó, cứ với x; y 2 F thì cũng sẽ có x C y 2 F và xy 2 F . Ta sẽ chứng minh quy nạp rằng : Với mỗi số nguyên dương n, các số thực dương a1; a2; :::; an và F là tập con của R thỏa mãn đồng thời các điều kiện dưới đây : 56 Tạp chí Epsilon, Số 20, 08/2021 .i / a2k2 F với mỗi chỉ số k, đồng thời a1 C a2 C ::: C a2021 2 F .i i / Nếu x; y 2 F; y ¤ 0 thì x y 2 F vàxy2 F Khi đó, ak 2 F với mỗi chỉ số k. Thật vậy trường hợp n D 1 là hiển nhiên. Giả sử khẳng định đúng tới n 2 N , ta xét n C 1 số thực dương a1; a2; :::; anC1 thỏa mãn : a2k2 F với mỗi chỉ số k, đồng thời a1Ca2C::akC1 2 F . Xét tập K xác định như sau : K D fx C a1y W x; y 2 F g Do tính chất của F nên với u; v 2 K ta sẽ có : u C v; u v; uv 2 K. Bây giờ giả sử u; v 2 K; v ¤ 0, ta viết v D x C a1y với x; y 2 F , ta xét các trường hợp sau : 1/ Nếu x a1y D 0 và y ¤ 0, khi đó a1 Dxy2 F nên v D x C a1y 2 F từ đó ta thấy là v2 F K do vậyuvD u:1v2 K: 1 2/ Nếu y D 0 thì v D x 2 F và x ¤ 0 nên1v2 F K để cũng có uv2 K. 3/ Nếu x a1y ¤ 0 thế thì m D x2 a21y2 2 F n 0 do giả thiết, từ đó sẽ kéo theo xm;ym2 F và có1vDxma1y m2 K kéo theouv2 K. Vậy, cứ với u; v 2 K thì u ˙ v 2 K; uv 2 K và khi v ¤ 0 thìuv2 K. Do giả thiết quy nạp và việc a2k2 F K,P 26k6nC1 ak D P 16k6nC1 ! ak –a1 2 K cho nên sẽ có được ak 2 K với mỗi k > 1. Ta viết a2 D s C a1t, với s; t 2 F và xét - Nếu t D 0, khi đó a2 D s 2 F nên P 2¤k ak 2 F và a2k2 F với mỗi chỉ số k, vì thế theo giả thiết quy nạp sẽ có ai 2 F với mọi chỉ số i từ 1 đến n C 1. - Nếu s D 0, thì a2 D ta1 với t 2 F và t > 0 nên X 16k6nC1 ak D .1 C t/ a1 CX 2 2 còn a1 2 F do .1 C t /a1 2 F và a1 D.t C 1/a1 a2 D ta1 2 F . 57 t C 1, từ a1 2 F và t 2 F ta cũng có Tạp chí Epsilon, Số 20, 08/2021 - Nếu st ¤ 0, ta có 2st 2 F n 0; a22; s; t; a21 2 F nên a1 Da22s2t2a21 Vì P 26k6nC1 ak D P 16k6nC1 ! ak 2st 2 F . a1 2 F , nên bộ .a2; a3; : : : ; anC1/ thỏa giả thiết quy nạp, nên có ak 2 F với mỗi chỉ số k > 1. Các luận xét vừa rồi, cho ta điều cần chứng minh. Bài 7 (KHTN 2020). Tìm tất cả các tập hợp A; B thỏa mãn đồng thời các điều kiện sau : i: A [ B D Z i i: Nếu x 2 A thì x 1 2 B i i i: Nếu x 2 B và y 2 B thì x C y 2 A Lời giải. Trường hợp 1. Nếu 0 2 B, từ (iii), với y D 0, ta suy ra với mọi x 2 B thì x 2 A. Cho nên : B A. Mà A [ B D Z, suy ra : A D Z. Kết hợp (ii), suy ra : B D Z. Trường hợp 2. Nếu 0 … B ) 0 2 A. Từ (ii), suy ra : 1 2 B Ta sẽ chứng minh, với 8k 2 N thì : 2k thuộc A nhưng không thuộc B; 2k C 1 thuộc B nhưng không thuộc A. Thật vậy, nếu 1 2 A ) 0 2 B (vô lý). Suy ra : 1 2 B; 1 … A. Giả sử khẳng định đúng n. Khi đó : Nếu 2k C 2 2 B, vì 1 2 B, suy ra : 2k C 1 2 A (vô lý) Do đó : 2k C 2 … B ) 2k C 2 2 A. Từ đó : 2k C 3 … A ) 2k C 3 2 B Vậy khẳng định đúng. Bây giờ, giả sử 9k0 < 0 mà 2k0 2 B. Chọn k sao cho jkj > jk0j ) 2k C 1 C 2k0 2 A (vô lý) Suy ra : 2k0thuộc A nhưng không thuộc B với mọi k0 nguyên âm. Chứng minh tương tư, 2k0 C 1 thuộc B nhưng không thuộc A, với mọi k0thuộc Z Vậy : .A; B/ D .Z; Z/, hoặc .A; B/ D .2Z; 2Z C 1/ Bài 8. Xét tập hợp A gồm các số nguyên dương, thỏa mãn đồng thời các điều kiện sau : (i) Số lớn nhất thuộc A là 100; (ii) Với mỗi số x thuộc A, mà x không phải là số nhỏ nhất thuộc A, luôn tồn tại ba số a; b; c (không nhất thiết đôi một khác nhau) thuộc A sao cho x D a C b C c. Hỏi tập A có tối đa bao nhiêu phần tử ? 58 Tạp chí Epsilon, Số 20, 08/2021 Lời giải. Xét tập A thỏa mãn các điều kiện của đề bài. Giả sử tập A có n phần tử, và các phần tử của A là a1; a2; :::; an. Vì 100 2 A (theo giả thiết), và 100 không chia hết cho 3 nên từ giả thiết (ii) suy ra phải có n > 3. Không mất tính tổng quát giả sử a1 < a2 < < an (1) Khi đó, theo giả thiết (i), ta được an D 100 (2) Với mỗi phần tử ak, 2 6 k 6 n, theo giả thiết (ii), tồn tại các số a; b; c (không nhất thiết đôi một khác nhau) thuộc A, sao cho : ak D a C b C c (3) và do 1 nên hiển nhiên phải có a; b; c 2 fa1; :::; ak1g (4) Từ đó, suy ra : a2 D a1 C a1 C a1 D 3a1 Giả sử với mọi k D 1; 2; :::; m, m là một số nguyên dương thỏa mãn 2 6 m < n, ta đã có : ak D lk:a1, trong đó lk là các số nguyên dương lẻ. Xét amC1. Theo 3 và 4, ta có : amC1 D x C y C z (5) trong đó x; y; z là các số (không nhất thiết đôi một khác nhau) thuộc tập hợp fa1; a2; :::; amg. Theo giả sử nêu trên, mỗi số x; y; z đều bằng một số lẻ lần a1, do đó, từ (5) suy ra : amC1 cũng bằng một lẻ lần a1. Vì vậy, theo nguyên lý quy nạp, với mọi k D 1; 2; :::; n, ak bằng một lẻ lần a1. Nghĩa là, với mọi k D 1; 2; :::; n, ta có : ak D lk:a1 trong đó lk là các số nguyên dương lẻ. Do 1 và 2, ta có : 1 D l1 < l2 < ::: < ln và 100 D ln a1 Từ đó, suy ra : n không vượt quá số số lẻ trong phạm vi từ 1 đến l, với l là ước ương lẻ lớn nhất của 100; suy ra : n > 13. Xét tập hợp A gồm các số : ak D .2k 1/:4; k D 1; 2; :::13 Rõ ràng số lớn nhất a13 D 100 và với k là số nguyên dương tùy ý thì : 59 Tạp chí Epsilon, Số 20, 08/2021 ak D .2k 1/ 4 D .2.k 1/ 1/ 4 C 4 C 4 D ak1 C a1 C a1: Cho nên tập A vừa xét thỏa các điều kiện đề bài. Vậy tập A có tối đa 13 phần tử. Bài 9. Cho S D f1; 2; :::; 10g. Giả sử tồn tại k > 2 tập hợp con A1; A2; :::; Ak của S sao cho : (i) jAij > 5;8i D 1; 2; ::; k; (ii) jAi \ Aj j > 2;8i ¤ j . Tìm giá trị lớn nhất của k. Lời giải. Ta đếm số bộ .a; ˚Ai; Aj˚/ thỏa mãn phần tử a thuộc hai tập con Ai; Aj theo hai cách. Cách thứ nhất : Với mỗi a 6 10 đặt fa là số tập chứa a. Khi đó, số bộ trên là P10 aD1 C2faDP10 aD1 f2a fa 2 Cách thứ hai : Với mỗi cặp ˚Ai; Aj có không quá 2 phần tử a cùng thuộc hai tập này. Do đó, số bộ không vượt quá 2Ckn D k2 k. Từ đó, ta có bất đẳng thức : 2k2 2k >P10 aD1 .f 2a fa/ Đặt S DP10 aD1 fa thì theo bất đẳng thức Cauchy-Schwarz, ta có : 2k2 2k >S2 10 S DS.S 10/ 10 Lại có S > 5k > 10 do mỗi tập hợp không có quá 5 phần tử. Vì thế nên 2k2 2k >5k.5k 10/ 10 (do S 10 > 5k 10, S > 5k > 0) dẫn tới k 6 6. Xét k D 6, ta xây dựng các tập như sau : A1 D f1; 2; 3; 4; 5g ; A2 D f1; 2; 6; 7; 8g ; A3 D f3; 4; 6; 8; 9g A4 D f1; 5; 6; 9; 10g ; A5 D f2; 4; 7; 9; 10g ; A6 D f3; 5; 7; 8; 10g Vậy giá trị lớn nhất cần tìm là k D 6. 60 Tạp chí Epsilon, Số 20, 08/2021 Bài 10 (USA MO 2001). Cho tập S Z thỏa mãn : (i) Tồn tại a; b 2 S sao cho .a; b/ D .a 2; b 2/ D 1 (ii) Nếu x; y 2 S thì x2 y 2 S Chứng minh rằng S D Z: Lời giải. Ta sẽ chứng minh 1 2 S. Khi đó ta có 0 2 S; 1 2 S; 2 2 S. Giả sử ta có m 2 S; 8jmj 6 n. Xét n C 1. Nếu n C 1 D t2thì jtj 6 n, suy ra t 2 S do đó n C 1 D t2 0 2 S. Nếu t2 < n C 1 < .t C 1/2thì n C 1 D t2 C m .0 < m 6 2t /, vì thế 0 < m 6 2t 6 t2 6 n nên m 2 S; m 2 S, hay n C 1 D t2 .m/ 2 S. Do vậy ta chỉ cần chứng miinh 1 2 S. Giả sử a; b; c 2 S khi đó : .a2 b2/ C c D a2 .b2 c/ ) .a2 b2/ C c 2 S Từ đó theo quy nạp ta suy ra n.a2 b2/ C c 2 S; 8n 2 N. Tương tự : n.a2 b/2 C c 2 S; 8n 2 Z Vậy n.a2 b2/ C c 2 S; 8n 2 Z Theo điều kiện (i) tồn tại a; b 2 S sao cho .a; b/ D .a 2; b 2/ D 1. Đặt A D a2 b2; B D a3.a 2/; C D b3.b 2/, suy ra B D .a2 a/2 a2; C D .b2 b/2 b2. Ta suy ra : c C mA C nB C pC 2 S; 8m; n; p 2 Z; c 2 S Đặt .A; B; C / D d. Nếu d > 1 suy ra tồn tại p nguyên tố và pjd ) a2 b2:::p mà .a; b/ D 1 suy ra hoặc .a C b/:::p hoặc .a b/:::p. Lại có a3.a 2/:::p ) a 2:::p (ngược lại a:::p ) b:::p mâu thuẫn với .a; b/ D 1). Tương tự b 2:::p ) .a 2; b 2/ > 1 mẫu thuẫn với điều kiện hai của bài toán. Vậy .A; B; C / D 1 suy ra tồn tại m; n; p sao cho mA C nB C pC D 1 c, hay 1 2 S. Từ đó, ta có điều phải chứng minh. 61 3. Bài tập tự luyện Tạp chí Epsilon, Số 20, 08/2021 Bài tập 1. Cho tập A khác rỗng là một tập con của ZC thỏa mãn đồng thời các điều kiện sau : (i) Với mọi a; b; c 2 A, ta có : .a; b; c/ D 1. (ii) Với mọi b; c 2 A, tồn tại a 2 A; a ¤ b; c sao cho : ajbc Chứng minh rằng tập hợp A có không quá 3 phần tử. Bài tập 2. Cho tập hợp X có n phần tử và m là tập con khác rỗng T1; T2; :::; Tm thỏa mãn : (i) jTij D 4; i D 1; m (ii) Với mọi i; j 2 X, tồn tại duy nhất k sao cho .i; j / Tk (iii) Với mọi 1 6 i < j 6 m thì jTi \ Tj j D 1 Tìm tất cả giá trị có thể của m; n. Bài tập 3. Xét các tập hợp A N thỏa mãn đồng thời các điều kiện sau : (i) jAj > 2 (ii) Với mọi a; b 2 AI a > b thìab a2 b22 A Tìm tất cả các tập hợp A như thế. Bài tập 4 (GGTH 2012). Trong chương trình Gặp gỡ toán học lần IV có tổng cộng 673 tựa sách và quyết định tổ chức đăng ký mua sách cho các thành viên tham gia. Sau khi thu phiếu đăng ký, ban tổ chức phát hiện ba điều thu vị sau : (i) Tất cả các bạn đều đăng ký mua đúng ba tựa sách và không có hai bạn nào mua cả ba tựa sách giống nhau. (ii) Hai bạn bất kỳ đăng ký mua giống nhau ít nhất một tựa sách. (iii) Không có tựa sách nào được tất cả các thành viên đăng ký mua. Chứng minh rằng ở kỳ Gặp gỡ toán học lần này có nhiều nhất 2011 bạn tham gia giao lưu và học tập. 62 Tạp chí Epsilon, Số 20, 08/2021 Gợi ý: Phát biểu lại bài toán dưới dạng ngôn ngữ tập hợp như sau : Cho tập A D f1; 2; :::; 673g và M D fA1; A2; :::; Ang là một họ các tập hợp con phân biệt của tập A thỏa mãn đồng thời : (i) jAij D 3; 8i D 1; 2; :::; n (ii) Ai \ Aj ¤ ;; 81 6 i < j 6 n (iii) A1 \ A2 \ ::: \ An D ; Chứng minh rằng jMj 6 2011: Bài tập 5 (Putnam 2016). Xét 2016 tập hợp A1; A2; :::; A2016 thỏa mãn đồng thời hai điều kiện : (i) jAij D 20; 8i D 1; 2; :::; 2016 (ii) jAi \ Aj j D 1; 81 6 i < j 6 2016 Chứng minh rằng A1 \ A2 \ ::: \ An ¤ ; Tài liệu tham khảo [1] Nguyễn Duy Liên, 23 bài toán số học tổ hợp hay và khó. [2] Group hướng tới Olympic toán Việt Nam. [3] Đề thi chọn đội tuyển học sinh giỏi cấp Tỉnh các năm [4] Tạp chí Pi - Số 3; tập 2 năm 2019: [5] Tạp chí toán học và tuổi trẻ. [6] https://artofprroblemsolving.com 63 Tạp chí Epsilon, Số 20, 08/2021 BẤT ĐẲNG THỨC VÀ CỰC TRỊ TRONG ĐỀ THI TUYỂN SINH VÀO LỚP 10 NĂM 2021 Trần Nam Dũng Thành phố Hồ Chí Minh GIỚI THIỆU Lâu nay tôi cũng thỉnh thoảng ngó nghiêng một chút các đề thi chuyên toán tuyển sinh 10 để soạn bài cho mấy cậu học trò. Nhưng là case by case thôi. Đợt này nhớ chiến dịch của Lê Phúc Lữ, tự dưng được làm quen với hàng loạt các đề thi mới và tôi bắt đầu để ý đến đặc điểm các đề thi. Tôi nhận thấy rằng bất đẳng thức và cực trị xuất hiện hầu hết trong các đề thi (có lẽ phải trên 80%). Xem kỹ hơn tôi thấy các bài này khá là khó (không biết có phải vì lâu ngày tôi không làm bất đẳng thức không). Thực sự nhiều bài có thể lấy làm đề thi VMO. Tuy nhiên, các đề khó nhưng lại không mới, cho nên sẽ vẫn có ai đó giải được, mà nhìn lời giải ngắn gọn chúng ta sẽ tưởng là các bài đó đơn giản. Thực tế thì một bài toán khó hay dễ không chỉ nhìn lời giải mà biết được, chúng ta phải xông vào làm thì mới biết. Vì thế tôi đã thử tập hợp lại các bài bất đẳng thức trong đề thi vừa rồi và thử giải. Thú thật là nhiều bài tôi lúng túng, phải tham khảo gợi ý của các bạn trong nhóm “Hướng tới Olympic toán VN”. Xin cảm ơn các bạn đó. Trong bài viết này, tôi sử dụng 36 bài toán thi của các trường và các tỉnh (các trường thi sau tôi không kịp đưa vào), giải và có những bình luận. Các ý kiến của tôi về các bài toán là chủ quan. Tôi sẽ rất vui nếu nhận được các lời giải, bình luận, ý kiến khác từ bạn đọc. Bản thân tôi luôn cho rằng các bài toán thi tuyển sinh 10 rất quan trọng. Khi ra một đề toán, chúng ta phải cân nhắc rất nhiều yếu tố: vừa có tính thách thức, vừa có tính gợi mở và đặc biệt là không đánh đố, đề thi nên có tính thẩm mỹ, gọn, đẹp. Trong 36 đề toán mà tôi trích dẫn có những đề hay, đẹp nhưng cũng có những đề quá xấu xí, cồng kềnh, gọi là “nhìn thấy đã không muốn làm”. Học tập rất cần cảm xúc, rất cần những bài toán đẹp, ý tưởng đẹp, lời giải đẹp. Đừng làm xấu xí toán học, làm cho nó trở nên “kinh khủng”. 1. Đề bài Bài toán 1 (Gia Lai). Cho x; y là các số thực không âm thỏa mãn điều kiện x C y D 2: Tìm giá trị lớn nhất và giá trị nhỏ nhất của biểu thức P D .5x2 C 7y/.5y2 C 7x/ C 151xy: 64 Tạp chí Epsilon, Số 20, 08/2021 Lời giải. Bài này cơ bản, phù hợp với thi tuyển sinh 10: Đặt t D xy và sử dụng điều kiện x C y D 2 ta tính được P theo t như sau P D 25t 2 10t C 280 D .5t 1/2 C 279: Đến đây thì dễ rồi. Bài toán 2 (Quảng Bình). Cho x; y; z 2 Œ5; 7: Chứng minh rằng pxy C 1 Cpyz C 1 Cpzx C 1 > x C y C z: Lời giải. Mẹo chính của bài này là x; y 2 Œ5; 7 suy ra jx yj 6 2: Từ đó 1 C xy >.x y/2 4C xy D.x C y/2 4: Dấu bằng xảy ra khi jx yj D 2: Vì là mẹo nên bài này khó bình luận. Bài toán 3 (Đại học Khoa học, Huế). Cho a; b; c là độ dài ba cạnh của một tam giác. Chứng minh rằng 3.ab C bc C ca/ 6 .a C b C c/2 < 4.ab C bc C ca/: Lời giải. Bài này thì quá cơ bản rồi. Vế trái chỉ là .a b/2 C .b c/2 C .c a/2 > 0: Còn vế phải thì dùng a2 < a.b C c/ là xong Bài toán 4 (Tây Ninh). Cho x; y; z là các số thực thỏa mãn 0 6 x; y; z 6 1: Tìm giá trị lớn nhất của biểu thức T D 2.x3 C y3 C z3/ .x2y C y2z C z2x/: Bài này là khá khó đối với học sinh THCS. Lời giải 1. Ta dự đoán giá trị lớn nhất là 3 đạt được khi x D y D z D 1. Để xuất hiện đánh giá liên quan đến x2y; y2z; z2x, ta sử dụng điều kiện để suy ra .1 x2/.1 y/ > 0: Suy ra x2y 6 1 x2 y: Cộng bất đẳng thức này và các bất đẳng thức tương tự, ta có x2y y2z z2x 6 3 .x2 C y2 C z2 C x C y C z/: Từ đó T D 2.x3 C y3 C z3/ x2y y2z z2x 6 2.x3 C y3 C z3/ C 3 .x2 C y2 C z2 C x C y C z/ D 3 x.1 x/.1 C 2x/ y.1 y/.1 C 2y/ z.1 z/.1 C 2z/ 6 3 Dấu bằng xảy ra khi x D y D z D 1. 65 Tạp chí Epsilon, Số 20, 08/2021 Lời giải. Trước hết ta chứng minh 2x3 x2y 6 2x xy. Thật vậy, điều cần chứng minh tương đương với 2x.1 x2/ xy.1 x2/ > 0 , x.1 x2/.2 y/ > 0: Tương tự 2y3 y2z 6 2y yz; 2z3 z2x 6 2z zx: Suy ra T 6 2.x C y C z/ .xy C yz C zx/: (1) Ta lại có .1 x/.1 y/ C .1 y/.1 z/ C .1 z/.1 x/ > 0: Suy ra 3 C xy C yz C zx 2.x C y C z/ > 0: (2) Từ (1) và (2) suy ra T 6 3. Dấu bằng xảy ra chẳng hạn khi x D y D z D 1. Lời giải 3. Dành cho các thầy cô giáo và các bạn học sinh THPT. Nếu để ý T là biểu thức thuần nhất bậc 3 T .ka; kb; kc/ D k3T .a; b; c/; ta sẽ thấy GTLN của T sẽ phải đạt được khi có một biến nào đó bằng 1. Chẳng hạn z D 1: Khi đó thì T D 2.x3Cy3C1/x2yy2x 6 x2CxCy2CyC31x2yy2x D 3.1x2/.1y/ 6 3: Vậy giá trị lớn nhất cần tìm là :3 Bài toán 5 (Bình Dương). Cho x; y; z > 0 thỏa mãn xy C yz C zx D 1: Chứng minh rằng: 10x2 C 10y2 C z2 > 4: Dấu bằng xảy ra khi nào? Lời giải. Ý tưởng là dùng AM-GM có trọng số. Các hệ số của x; y giống nhau nên x; y có vai trò như nhau. Ta tách 10 a22 x2 C 10 a22 y2 Ca2x2 C z2 2Ca2y2 C z2 2> .20 a2/xy C axz C ayz: Ta chọn a sao cho vế trái là hằng số, tức là 20 a2 D a: Giải ra được a D 4: Dấu bằng xảy ra khi và chỉ khi x D y và 4x D 4y D z; tức là khi x D y D 13; z D 13: Bài này được tính là khá thách thức. Bài toán 6 (Cần Thơ). Cho x; y; z là các số thực dương. Chứng minh rằng .x C 2/2 y C zC.y C 2/2 z C xC.z C 2/2 x C y> 12: 66 Tạp chí Epsilon, Số 20, 08/2021 Lời giải. Sử dụng bất đẳng thức Cauchy-Schwarz ở dạng phân thức, ta có V T >.x C y C z C 6/2 2.x C y C z/ : Tiếp tục áp dụng AM-GM cho hai số x C y C z và 6 thì .x C y C z C 6/2 > 4.x C y C z/:6 D 24.x C y C z/: Suy ra điều phải chứng minh. Nếu quen cách dùng bất đẳng thức Cauchy-Schwarz như ở trên thì cũng ở dạng cơ bản. Bài toán 7 (Tiền Giang). Cho a; b; c là các số thực dương thay đổi thỏa mãn abc D 1: Tìm giá trị lớn nhất của biểu thức M D1 a2 C 2b2 C 3C1 b2 C 2c2 C 3C1 c2 C 2a2 C 3: Lời giải. Bài này khá khó chịu nếu không liên hệ được với bài toán quen thuộc sau: Cho abc D 1 khi đó ab C b C 1C1 bc C c C 1C1 1 ca C a C 1D 1: Ta có a2 C 2b2 C 3 D .a2 C b2/ C .b2 C 1/ C 2 > 2.ab C b C 1/: Từ đó1 a2 C 2b2 C 161 2.ab C b C 1/: Từ đó GTLN cần tìm bằng 12; đạt được khi a D b D c D 1: Bài toán 8 (Quảng Nam). Cho ba số thực dương x; y; z thỏa mãn xy C yz C zx D xyz: Tìm giá trị nhỏ nhất của biểu thức 9z C zx2Cy2 H Dx2 9x C xy2Cz2 9y C yz2: Lời giải. Ý đầu tiên để giải bài này là đưa điều kiện về 1x C 1y C 1z D 1 rồi đặt a D 1x; b D 1y; c D 1z: Điều kiện a C b C c D 1 dĩ nhiên là dễ sử dụng hơn. Lúc này thì 9b2 C 1Cb H Da 9c2 C 1Cc 9a2 C 1: Ta dùng AM-GM để đánh giá các số hạng của H như sau 9b2 C 1Da C 9ab2 9ab2 9b2 C 1D a 9ab2 a 67 9b2 C 1> a 9ab2 6b D a 3ab2: Tạp chí Epsilon, Số 20, 08/2021 Cộng bất đẳng thức này và các bất đẳng thức tương tự, ta suy ra (ở đây ta dùng bất đẳng thức quen thuộc 3.ab C bc C ca/ 6 .a C b C c/2) H > a C b C c 32.ab C bc C ca/ > 1 32 13D12: Dấu bằng xảy ra khi a D b D c D 13, tức là khi x D y D z D 3. Bài toán 9 (Ninh Thuận). Cho các số thực dương x; y; z thỏa mãn xyz D 18: Chứng minh rằng 1 xy C yz C zx x C y C z623: 1 Lời giải. Bài này nhẹ nhàng. Ý tưởng là sử dụng đánh giá trung gian để đưa về một biến. Đặt x C y C z D6t2: Sử dụng đánh giá .xy C yz C zx/2 > 3xyz.x C y C z/; ta sẽ suy ra .xy C zy C zx/2 >94t 2: Suy ra xy C yz C zx >32t : Từ đó x C y C z62t3t26D2316.t 2/2 623: 1 xy C yz C zx 1 Lưu ý, cách đặt chỉ giúp chúng ta làm việc với các số đẹp hơn, không phải bí quyết của cách giải. Nếu ta đặt x C y C z D t cũng được. Bài toán 10 (Nghệ An). Cho các số dương a; b; c thỏa mãn ab C bc C ca 6 3abc: Tìm giá trị nhỏ nhất của biểu thức P Dpa C b Cpb C c Cpc C a sa2 C b2 2.a C b/ s b2 C c2 2.b C c/ s c2 C a2 2.c C a/: Lời giải. Bài này là rất khó đối với mức THCS. Đã phân thức lại còn chứa căn, có điều kiện, lại cồng kềnh, bốn thứ gộp lại. Để giải bài toán, ta cần vài đánh giá để đơn giản bớt biểu thức P: Trước hết, ta có bất đẳng thức phụ q 2 6 a C b pab: a2Cb2 q a2Cb2 aCb pab rồi nhân Bất đẳng thức này thu được bằng cách viết lại thành lượng liên hợp. 2 2 6aCb 2 Từ đây suy ras pa C bDpa C b sab 2.a C b/ 6a C b pab a2 C b2 68 a C b: Tạp chí Epsilon, Số 20, 08/2021 Từ đó P > s ab a C bC s bc b C cC rca c C a: Đến đây thì dễ thở hơn rồi. Đặt x D 1a; y D 1b; z D 1cthì x C y C z 6 3 và ta cần giá trị nhỏ nhất của Q D s 1 x C yC s 1 y C zC r1 z C x: Áp dụng bất đẳng thức Cauchy-Schwarz ta có Q >9 px C y Cpy C z Cpz C x>9 p3 6D3p2: p3.x C y C y C z C z C x/>9 Dấu bằng xảy ra khi x D y D z D 1 tức là khi a D b D c D 1. Vậy giá trị nhỏ nhất của P là p32đạt được khi a D b D c D 1. Bài toán 11 (Bà Rịa – Vũng Tàu). Xét các số thực không âm a; b; c thỏa mãn điều kiện a2 C b2 C c2 D 1: Tìm giá trị lớn nhất và giá trị nhỏ nhất của biểu thức 1 C bc Cb S Da 1 C caCc 1 C ab : Lời giải. Bài này rất khó, chỉ một trong hai giá trị cần tìm đã là đủ khó rồi. Có thể “đoán” được giá trị nhỏ nhất là 1 và giá trị lớn nhất là p2 (bài toán gốc là như vậy), nhưng xử lý tiếp thế nào? Ý tưởng chung là tìm cách “quy đồng mẫu số” bằng một đánh giá trung gian. Ở chiều giá trị lớn nhất, ta sẽ chứng minh rằng 1 C bc 6ap2 a a C b C c(3) Điều này sẽ luôn đúng khi a D 0 còn với a > 0 thì tương đương với .a C b C c/2 6 2.1 C bc/2: Khai triển ra và áp dụng giả thiết, điều này tương đương với 2ab C 2ac 6 a2 C b2 C c2 C 2b2c2 C 2bc , .a b c/2 C 2b2c2 > 0: Dấu bằng xảy ra khi a D b C cvà bc D 0. Từ đây ta tìm được giá trị lớn nhất của S là p2 đạt được, chẳng hạn khi a D b D p12; c D 0. Ở chiều giá trị nhỏ nhất, ta lại phải dùng một đánh giá khác, chú ý rằng a 6 1 nên a Cabc 6 a Ca.b2 C c2/ 2D a Ca.1 a2/ 2D a Ca.1 C a/.1 a/ 26 a C1 2.1 a/ 2D 1: 1Cbc D a2 Cho nên a aCabc > a2. Cộng các bất đẳng thức tương tự lại ta có S > a2 C b2 C c2. Dấu bằng xảy ra, chẳng hạn khi a D 1; b D c D 0: 69 Tạp chí Epsilon, Số 20, 08/2021 Bài toán 12 (Quảng Trị). Cho a; b; c là các số thực tùy ý (a) Chứng minh rằng 4.a2 ab C b2/ > .a C b/2: (b) Chứng minh rằng 4.a2 C b2/.b2 bc C c2/.3c2 C 2ca C 3a2/ > .a C b/2.b C c/2.c C a/2: Lời giải. Bài này rất cơ bản. Câu (b) cũng cứ tách ra từng nhóm mà làm. Bài toán 13 (Thái Bình). Cho a; b; c là các số thực dương thỏa mãn a2 C b2 C c2 D 3abc: Tìm giá trị lớn nhất của biểu thức 3a2 C 2b2 C c2Cb T Da 3b2 C 2c2 C a2Cc 3c2 C 2a2 C b2: Lời giải. Ta dùng AM-GM để đánh giá mẫu số 3a2 C 2b2 C c2 > 6p6a6b4c2 D 6ap3b2c: Từ đóa 3a2 C 2b2 C c261 6p3b2c: Cộng bất đẳng thức này và các bất đẳng thức tương tự cho hai số hạng còn lại, ta có T 616 1 p3b2cC1 p3c2aC1 p3a2b Tiếp tục dùng AM-GM ta có2 bC1c>3 p3b2c: : (4) Tương tự2 p3c2a;2aC1b>3 p3a2b: Suy ra1 cC1a>3 p3b2cC1 p3c2aC1 p3a2b(5) Kết hợp (4) và (5), ta được aC1bC1c>1 T 616 1aC1bC1c D16 ab C bc C ca abc 616 a2 C b2 C c2 abc D12: Dấu bằng xảy ra khi a D b D c D 1. Đây là một bài toán khá khó. Bài toán 14 (Quảng Ninh). Cho hai số thực x; y: thỏa mãn 0 < x < y 6 8 và xy 6 4x C 3y: Chứng minh x2 C y2 6 100: 70 Tạp chí Epsilon, Số 20, 08/2021 Lời giải. Đây là bài toán khó và đẹp. Trước hết, để ý rằng nếu x < 6 thì hiển nhiên là x2 C y2 < 62 C 82 D 100; nên tiếp theo ta chỉ cần xét trường hợp x > 6. Ta khai thác điều kiện x < y 6 8 và xy 6 4x C 3y; .8 x/.8 y/ > 0. Suy ra xy C 64 > 8x C 8y, suy ra 4x C 3y C 64 > 8x C 8y. Từ đó 4x C 5y 6 64. Vì 6 6 x < y 6 8 nên y x 6 2. Suy ra .y x/2 6 2.y x/. Từ đây suy ra x2 C y2 6 2xy C 2.y x/ D 8x C 6y C 2.y x/ D 6x C 8y D32.4x C 5y/ Cy2632 64 C82D 100: Chứng minh hoàn tất. Bài toán 15 (Lào Cai). (a) Cho hai số thực dương x; y thỏa mãn x C y 623: Tìm giá trị nhỏ nhất của A D 53x C 53y C1x2C1y2: (b) Cho ba số thực dương x; y; z thỏa mãn x2 C y2 C z2 > 3: Chứng minh rằng x4 C y4 C z4 C x3 C y3 C z3 > 3 C x C y C z: Lời giải. Hai bài này đều khá cơ bản. (a) Ta dự đoán điểm rơi là 13;13nên sẽ dùng AM-GM tương ứng với điểm rơi này: 27x C 27x C1x2> 3p3272 D 27: Hay là 54x C 1x2 > 27. Tương tự 54y C 1y2 > 27: Từ đó A D 54x C1x2C 54y C1y2 .x C y/ > 27 C 27 23D1603: (b) Áp dụng bất đẳng thức AM-GM ta có x4 C y4 C z4 C 3 D .x4 C 1/ C .y4 C 1/ C .z4 C 1/ > 2.x2 C y2 C z2/ > 6: Suy ra x4 C y4 C z4 > 3. Lại áp dụng AM-GM ta có x3 C y3 C z3 C x C y C z > 2.x2 C y2 C z2/ > x2 C y2 C z2 C 3 > 2.x C y C z/: Suy ra x3 C y3 C z3 > x C y C z. 71 Tạp chí Epsilon, Số 20, 08/2021 Bài toán 16 (Khánh Hòa). Cho các số thực x1; x2; : : : ; x21 thỏa mãn x1; x2; : : : ; x21 > 2 và x31 C x32 C C x321 D 12: Chứng minh rằng x1 C x2 C C x21 6 18: Lời giải. Bài này tuy kỹ thuật nhẹ nhàng nhưng có thể xếp vào bài toán khó. Ý tưởng là ta tìm cách đánh giá xi qua x3ibằng các biểu thức hiển nhiên dương. Biểu thức thứ nhất hiển nhiên là xi C 2 (từ điều kiện xi > 2). Còn biểu thức thứ hai là một bình phương dạng .xi C a/2. Để chỉ xuất hiện x3ivà xita chọn a D 1: Vậy là ta dùng .xi C 2/.xi 1/2 > 0: Suy ra x3i 3xi C 2 > 0, hay x3i C 2 > 3xi. Cho i D 1; 2; : : : ; 21 rồi cộng lại, ta được 12 C 42 > 3.x1 C x2 C ::: C x21/: Suy ra x1 C x2 C C x21 6 18: Chứng minh hoàn tất. Bài toán 17 (PTNK, ĐHQG TP HCM). Cho dãy n số thực x1; x2; : : : ; xn .n > 5/ thỏa mãn điều kiện x1 6 x2 6 6 xn và x1 C x2 C C xn D 1: Chứng minh rằng (a) Nếu xn >13thì x1 C x2 6 xn: (b) Nếu xn 623thì tồn tại số nguyên dương k < n; sao cho 36 x1 C x2 C C xk 623: 1 Lời giải. Tương tự như bài trên, bài này kỹ thuật nhẹ nhàng nhưng là bài toán khó. Và hai ý về cơ bản là không liên quan đến nhau. (a) Giả sử ngược lại x1 C x2 > xn >13. Khi đó x2 > 0 và vì thế xn > 0 với mọi n > 2. Lúc này x1 C x2 C C xn D .x1 C x2/ C .x3 C x4/ C .:::/ C xn >13C13C13D 1: Mâu thuẫn. Vậy điều giả sử là sai và ta có x1 C x2 6 xn. (b) Xét hai trường hợp. Trường hợp thứ nhất xn >13. Khi đó thì x1 C x2 C C xn1 D 1 xn; sẽ 623và >13và ta có k D n 1 là số nguyên dương cần tìm. 72 Tạp chí Epsilon, Số 20, 08/2021 Trường hợp thứ hai, xn <13. Khi đó xk <13với mọi k. Vì x1 C x2 C C xn D 1; nên sẽ tồn tại số nguyên dương k nhỏ nhất sao cho x1 C x2 C C xk >13: Ta chứng minh với số k này Thật vậy, nếu ngược lại x1 C x2 C C xk 623: x1 C x2 C C xk >23; thì x1 C x2 C C xk1 D .x1 C x2 C ::: C xk/ xk >2313D13; mâu thuẫn với cách chọn k: Lưu ý, ở câu (a) có một cái bẫy. Nếu chứng minh trực tiếp mà sử dụng đánh giá 2 3> x1 C x2 C C xn1 > x1 C x2 C x3 C x4 > 2.x1 C x2/; rồi suy ra x1 C x2 613là ta phạm sai lầm ở dấu bất đẳng thức thứ hai: Ở đây xilà số thực và có thể âm. Bài toán 18 (Lâm Đồng). Cho a; b; c là các số dương và a C b C c D 6: Tìm giá trị nhỏ nhất của biểu thức P Da3 a2 C 4ab C b2Cb3 b2 C 4bc C c2Cc3 c2 C 4ca C a2: Lời giải. Bài này sử dụng một đánh giá trung gian, xem hướng dẫn ở bài 21: Bài toán 19 (Hà Nam). Cho ba số thực dương x; y; z thỏa mãn x C y C z 6 1: Chứng minh 1 x2 1 1 y2 1 1 z2 1 > 512: Lời giải. Sử dụng điều kiện x C y C z 6 1 ta có x2 1 >.x C y C z/2 1 Từ đó chỉ cần chứng minh x2 1 D.2x C y C z/.y C z/ x2: .2x C y C z/.y C z/.2y C z C x/.z C x/.2z C x C y/.x C y/ > 512x2y2z2: Cái cuối này dùng AM-GM là ra. 73 Tạp chí Epsilon, Số 20, 08/2021 Thú vị là bất đẳng thức này “tương đương” với bất đẳng thức IMO 2001 nổi tiếng: Cho a; b; c là các số thực dương, khi đó a pa2 C 8bcCb pb2 C 8caCc pc2 C 8ab> 1: Các bạn thử thiết lập sự tương đương đó nhé! Đương nhiên, tuy là “tương đương” nhưng bất đẳng thức IMO khó hơn. Bài toán 20 (Daklak). Cho các số thực dương a; b; c thỏa mãn a C b C c 6 2: Tìm giá trị nhỏ nhất của biểu thức P Db.a2 C 1/2 a2.b2 C 1/ Cc.b2 C 1/2 b2.c2 C 1/ Ca.c2 C 1/2 c2.a2 C 1/: a; y D b2C1 Lời giải. Đặt x D a2C1 b; z D c2C1 c; thì yCy2 P Dx2 Áp dụng bất đẳng thức Cauchy-Schwarz ta có P Dx2 yCy2 zCz2 zCz2 x: x> x C y C z D a C b C c C1aC1bC1c> a C b C c C9 a C b C c D a C b C c C4 a C b C c C5 a C b C c> 4 C52D132: Dấu bằng xảy ra khi a D b D c D 23: Vậy giá trị nhỏ nhất của P là 132: Bài này không khó nhưng phát biểu cồng kềnh, xấu xí, là ghép nối cơ học của hai bài toán. Bài toán 21 (Bình Phước). Cho a; b; c là các số dương (a) Chứng minh rằng (b) Chứng minh rằng a3 a2 C b2> a b2: a2 C ab C b2Cb3 a3 b2 C bc C c2>a C b C c 3: Lời giải. Ý (a) tất nhiên là không khó. Ta chỉ cần biến đổi tương đương. a3 a2 C b2> a b2, 2a3 > 2a3 C 2ab2 ba2 b3 , b.a b/2 > 0: Hơn nữa, đây là gợi ý quan trọng để giải ý (b) (cũng là gợi ý để giải các bài 19: 74 Tạp chí Epsilon, Số 20, 08/2021 Thật vậy, do a2 C ab C b2 632.a2 C b2/; nêna3 a2 C ab C b2>23 a b2 D2a3b3: Cộng bất đẳng thức này và các bất đẳng thức tương tự cho hai số hạng còn lại, ta suy ra bất đẳng thức cần chứng minh. Bài toán 22 (Quảng Ngãi). Cho các số thực a; b; c đôi một khác nhau và thỏa mãn điều kiện .c C a/.c C b/ D 4: Chứng minh rằng .a b/2 C1 1 .c C a/2 C1 .c C b/2 > 1: Đây là một bài toán khá khó đối với trình độ THCS. Lời giải. Đặt x D c C a; y D c C b thì xy D 4 và .a b/2 D .x y/2; thì x ¤ y và xy D 4: Ta quy về bài toán chứng minh 1 .x y/2 C1x2C1y2> 1: (6) Không mất tính tổng quát có thể giả sử x; y > 0. Ta chỉ cần chứng minh .x y/2 Cxyx2Cxyy2> 4: (7) xy Đặt t D xy ¤ 1, bất đẳng thức cuối cùng này tương đương với t .t 1/2 C t C1t> 4; hay là hoặc t2 C t2.t 1/2 C .t 1/2 > 4t .t 1/2; t4 6t 3 C 11t 2 6t C 1 > 0; t2 6t C 11 6tC1t2> 0: (8) Đặt u D t C 1t; thì (8), tương đương u2 2 6u C 11 > 0 , .u 3/2 > 0: Vậy ta có điều phải chứng minh. Kỹ thuật trong bài này khá hay. Phép đặt x D c C a; y D c C b là khá tự nhiên. Bước chuyển từ (6)) sang (7) là bước “thuần nhất hóa”, còn bước đặt t D xylại là bước “chuẩn hóa”. 75 Tạp chí Epsilon, Số 20, 08/2021 Bài toán 23 (Đắk Nông). Cho hai số thực a; b thỏa mãn a; b 2 Œ2021; 2022: Tìm giá trị lớn nhất của biểu thức A D .a C b/ 2021 aC2021 b : Lời giải. Bài này cơ bản. Việc đưa các số 2021; 2022 vào chỉ làm rối thêm, bớt đẹp. Cứ để a; b thuộc Œ1; 2 và tìm GTNL của .a C b/ thì chân phương hơn. Ta chỉ cần tìm giá trị lớn nhất của 1 1 aC1b ; .a C b/ aC1b D 2 Ca2 C b2 ab : Ta có .2022a 2021b/.2022b 2021a/ > 0 nên từ đây suy ra 2021 2022.a2 C b2/ 6 .20212 C 20222/ab; 2021 2022.a2 C b2/ 6 .20212 C 20222/ab: Suy ra ab 620212 C 20222 a2 C b2 Từ đó tìm được giá trị lớn nhất cần tìm là 2021 2022: 2021 2022D 6064 C20212 2022: Lời giải hoàn tất. 4042 C2021.20212 C 20222/ Bài toán 24 (Hòa Bình). Cho các số thực dương x; y; z thỏa mãn x C y C z D 4: Chứng minh xyC1xz> 1: 1 Lời giải. Bài này cơ bản, phù hợp đề thi tuyển sinh 10: Ta chỉ cần áp dụng AM-GM hai lần 1 xyC1xz>4 xy C xzD4 x.y C z/ >4 2 D 1: Chứng minh hoàn tất. xCyCz 2 Bài toán 25 (Vĩnh Long). Cho số thực x thỏa mãn 1 6 x 6 2: Tìm giá trị lớn nhất và giá trị nhỏ nhất của biểu thức T D3 C x xC6 x 3 x: 76 Tạp chí Epsilon, Số 20, 08/2021 Lời giải. Bài này cơ bản, đẹp, phù hợp với đề tuyển sinh 10: Ta có T D 1 C3xC 1 C3 3 xD 2 C9 x.3 x/: Để tìm min, max của T ta chỉ cần tìm min, max của x.3 x/. Ta có x.3 x/ D94 x 32 2694: Dấu bằng xảy ra khi x D 32: Mặt khác, do 1 6 x 6 2 nên 12 6 x 32 612: Suy ra x 32 2614: Suy ra x.3 x/ D94 x 32 2>9414D 2: Dấu bằng xảy ra khi x D 1 hoặc x D 2: Từ đó giá trị lớn nhất của T bằng 2 C 92 D 132đạt được khi x D 1 hoặc x D 2. Giá trị nhỏ nhất của T bằng 2 C 4 D 6, đạt được khi x D 32: Bài toán 26 (Kiên Giang). Cho x; y; z là các số thực lớn hơn 2021 và thỏa mãn xC1yC1zD2 1 Chứng minh rằng ta có bất đẳng thức sau 2021: px C y C z >px 2021 Cpy 2021 Cpz 2021: 2021 ; b Dy Lời giải. Đặt a D x 2021 ; c D z 2021 thì a; b; c > 1 và 1a C 1b C 1c D 2. Ta chứng minh pa C b C c >pa 1 Cpb 1 Cpc 1: Đây là bài toán thi Olympic của Iran. Cách giải là dùng Cauchy-Schwarz ở dạng p.x C y C z/.a C b C c/ >pxa Cpyb Cpzc; cho các số a1 a;b1 b;c1 cvà a; b; c với chú ý a1 a C b1 b C c1 c D 1. Bài toán của Iran rất đẹp, nhưng bài toán chế biến này thì không còn đẹp nữa. Bản thân tôi không thích kiểu “sáng tác” này cho lắm. Bài toán 27 (Bình Định). Cho các số thực x; y: Tìm giá trị lớn nhất và giá trị nhỏ nhất của biểu thức T Dx y x4 C y4 C 6: Lời giải. Bài này cơ bản. Gợi ý .x y/2 6 2.x2 C y2/ và x4 C y4 >.x2Cy2/2 2: 77 Tạp chí Epsilon, Số 20, 08/2021 Bài toán 28 (Bình Định). Cho x; y; z là ba số dương thỏa x C y C z D 1: Chứng minh rằng P D1 x2 x C yzC1 y2 y C zxC1 z2 z C xy> 6: Lời giải. Sử dụng thuần nhất hóa 1 x2 D .x C y C z/2 x2 D .2x C y C z/.y C z/; x C yz D x.x C y C z/ C yz D .x C y/.x C z/: Từ đó nếu đặt a D y C z; b D z C x; c D x C y; thì V T D.b C c/a bc C.c C a/b caC.a C b/c ab DbaCabCcaCacCbcCcb: Chứng minh hoàn tất. Bài toán 29 (Cà Mau). Cho a; b là hai số thực dương sao cho pa Cpb D 1: Chứng minh p3a C b Cp3b C a 6 2p3a C b p3b C a: Lời giải. Đây là một bài toán đẹp. Cách giải cũng khá chân phương, giống như giải phương trình chứa căn vậy. Áp dụng bất đẳng thức Cauchy-Schwarz cho hai căn thức vế trái, ta có p3a C b Cp3b C a 6p2.3a C b C 3b C a/ Dp8a C 8b: Bất đẳng thức cần chứng minh sẽ đúng nếu ta chứng minh được p8a C 8b 6 2p.3a C b/.3b C a/: Ta biến đổi tương đương p8a C 8b 6 2p.3a C b/.3b C a/; hay là 2.a C b/ 6 .3a C b/.3b C a/; hoặc 3.a C b/2 C 4ab 2.a C b/ > 0: (9) Đặt ab D t2: Theo giả thiết pa Cpb D 1; suy ra a C b C 2t D 1; tức là a C b D 1 2t: Thay vào (9), ta cần chứng minh 3.1 2t /2 C 4t 2 2.1 2t / > 0; hay 16t 2 8t C 1 > 1 , .4t 1/2 > 0: Dấu bằng xảy ra khi và chỉ khi t D 14; từ đây tính được a D b D 14: 78 Tạp chí Epsilon, Số 20, 08/2021 Bài toán 30 (Bình Định). Cho a; b là các số dương thỏa mãn a C 2b > 3: Tìm giá trị nhỏ nhất của biểu thức P D3a2 C a2b C 9ab2 2 C .8 C a/b3 ab : Lời giải. Bài này thật xấu xí. Không hiểu tại sao lại phát biểu một bài toán xấu như vậy. Và đây là một bài khá khó. Gợi ý điểm rơi là a D 32; b D 34: Bài toán 31 (Thanh Hóa). Cho ba số thực dương x; y; z thỏa mãn x C y C z D 1: Tìm giá trị nhỏ nhất của biểu thức P D 15p3.x4 C y4 C z4/ Cxy C yz C zx x2y C y2z C z2x: Lời giải. Bài này cơ bản nhưng dùng nhiều đánh giá trung gian. Gợi ý p3.x4 C y4 C z4/ > x2 C y2 C z2; và 3.x2y C y2z C z2x/ 6 .x2 C y2 C z2/.x C y C z/: Bài toán 32 (Hải Phòng). Cho các số thực dương x; y; z: Chứng minh rằng p2y C zCzpzx p2x C yCypyz xpxy p2z C x>p3xyz: Lời giải. Bài này khá cơ bản đối với các bạn quen sử dụng AM-GM và Cauchy-Schwarz để đánh giá. Chia hai vế của bất đẳng thức cho p3xyz ta đưa bất đẳng thức cần chứng minh về dạng p3z.2x C y/Cy p3x.2y C z/Cz x p3y.2z C x/> 1: Áp dụng bất đẳng thức AM-GM cho mẫu thức, ta có p3z.2x C y/ 63z C 2x C y 2; cùng các bất đẳng thức tương tự. Do đó V T >2x 3z C 2x C yC2y 3x C 2y C zC2z 3y C 2z C x: Áp dụng bất đẳng thức Cauchy-Schwarz X 2x 3z C 2x C y>.2x C 2y C 2z/2 2x.3z C 2x C y/ C 2y.3x C 2y C z/ C 2z.3y C 2z C x/ D 1: Từ đó suy ra điều cần chứng minh. 79 Tạp chí Epsilon, Số 20, 08/2021 Bài toán 33 (Yên Bái). Cho a; b; c là các số thực dương thỏa mãn 18abc D a C 2b C 3c: Chứng minh .1 C a2/.1 C 4b2/.1 C 9c2/ > 8: Lời giải. Bài này cơ bản. Đặt x D a; y D 2b; z D 3c thì 3xyz D x C y C z ta cần chứng minh .1 C x2/.1 C y2/.1 C z2/ > 8: Áp dụng bất đẳng thức AM-GM ta có 3xyz D x C y C z > 3p3 xyz: Từ đây suy ra xyz > 1: Lại áp dụng AM-GM thì ta có .1 C x2/.1 C y2/.1 C z2/ > 2x 2y 2z D 8xyz > 8: Chứng minh hoàn tất. Bài toán 34 (Ninh Bình). Cho x; y; z là các số thực dương thỏa mãn 1 xCy C 1 yCz C 1 zCx D 12: Tìm giá trị lớn nhất của biểu thức 2x C 3y C 3zC1 P D1 Lời giải. Sử dụng bất đẳng thức 1 3x C 2y C 3zC1 3x C 3y C 2z: aC1bC1cC1d>16 a C b C c C d; cho các biểu thức ở vế trái. Bài toán 35 (Phú Thọ). Cho ba số dương x; y; z: Tìm giá trị nhỏ nhất của biểu thức y2 C yzCy2 px Cpz P Dxz xz C yzC 2px: Lời giải. Đây là một bài toán khó. Lời giải thông qua vài phép đánh giá, rồi dự doán điểm rơi. Đặt a D xy; b Dyzthì xz D ab và P Da b C 1Cb a C 1C1 2pabC12D 1 Ca b C 1C 1 Cb a C 1C1 1 b C 1C1 C1 2pab32>4.a C b C 1/ 2pab32 D .a C b C 1/ Đặt t D a C b thì a C 1 a C b C 2C1 a C b 3 2: 2t .t C 2/ D.t 2/2 2t .t C 2/ Dt2 4t C 4 t C 2C1t72D8t .t C 1/ C 2.t C 2/ 7t .t C 2/ P 2 >4.t C 1/ Suy ra P > 2: Dấu bằng xảy ra khi t D 2; tức a D b D 1; hay x D y D z: 80 2t .t C 2/ > 0: